Производная дроби формула: Производная дроби, формула и примеры

Содержание

Производная th. Производная сложной функции

Эпиграф: Однажды спросила: “Чем производная отличается от произведения?” “Производную изучают на уроке математики, а произведение – на уроке литературы”, – последовал ответ ученика.

В эпиграфе описана реальная ситуация из моей практики. Вопрос возник, когда ученик запутался в правилах дифференцирования функций, в частности, не смог определить производную произведения двух функций. Во избежание подобной трактовки этой статьи напомню, что мы занимаемся именно математикой, и здесь термин “произведение” обозначает результат операции умножения, а “производная” это предел отношения приращения функции к приращению аргумента, когда последнее стремится к нулю. Процесс вычисления производной называется дифференцированием.

Производные элементарных функций по определению, т.е. через предел, вычисляют только однажды на лекции (на уроке), чтобы закрепить связь производной и предела.

В дальнейшем нас интересует только практическое применение этого понятия, поэтому для вычисления производной пользуются готовыми Формулами и Правилами дифференцирования функций.

Здесь мы посмотрим как надо и как не надо вычислять производные, но, к сожалению, многие школьники и даже студенты это делают.

Как надо вычислять производные

Об этом написано везде, во всех учебниках и на множестве сайтов в сети.
Чтобы находить производные, нужно, пользуясь тем или иным источником, всё-таки выучить Формулы дифференцирования элементарных функций. Например, посмотрите подробную статью о Для более сложных, чем табличные, комбинированных функций применяются правила вычисления производной суммы, произведения, дроби. Соответствующие математические выражения также можно найти где угодно. Но, на мой взгляд,
Правила
дифференцирования функций лучше формулировать и заучивать словами:
  1. Постоянный множитель можно выносить за знак производной.
  2. Производная суммы равна сумме производных.
  3. Производная произведения равна “производная первого сомножителя, умноженная на второй, плюс производная второго сомножителя, умноженная на первый”.
  4. Производная дроби равна “производная числителя, умноженная на знаменатель, минус производная знаменателя, умноженная на числитель, деленные на знаменатель в квадрате”.
  5. Производная сложной функции равна производной внешней функции, умноженной на производную внутренней, и вычисляется “с продолжением” до табличной.
Последнее правило самое трудное для применения. Здесь допускается большое количество ошибок, поэтому о нём подробнее ниже.

Как НЕ надо вычислять производные

  1. Прежде всего, не надо усложнять простое.
  2. Не надо путать степенную x а и показательную a x функции.

В большинстве последующих примеров представлены варианты вычислений производных, в которых

1.

вычисления выполнены совсем плохо , с явными ошибками;
2. правильно, но неоптимально , т.е. долго и с вероятными ошибками на невнимательность;
3. совсем хорошо .

  • Не надо усложнять простое.
  • Обратите внимание, на правило, которое я поставила под номером один.

    Если в произведении один из сомножителей является постоянной величиной, то совершенно не обязательно пользоваться правилом производной произведения. Более того, не нужно этого делать, так как часто такая операция сопровождается ошибками.

    Пример 1.

    Если в дроби числитель или знаменатель является постоянной величиной, то совершенно необязательно пользоваться правилом для производной дроби. Это действие у школьников и студентов ещё чаще сопровождается ошибками.

    Постоянный множитель можно выносить за знак производной!

    Пример 2.

    Пример 3.


    Самая частая ошибка в подобных примерах – забыть поставить штрих (обозначение производной) над числом или поставить его и “не увидеть” при следующем действии, т. е. не учесть, что производная константы (числа) равна нулю.

    Здесь для первого и третьего примеров простота и качество подхода c вынесением числового множителя за скобки очевидна. Но не всё так однозначно для второго примера, где в знаменателе находится тригонометрическая функция. Более того, соглашусь, что для тех учеников, которые плохо владеют производной сложной функции (правилом 5), более предпочтительным в этом примере может оказаться правило дифференцирования дроби.

    Однако, для ряда других функций, особенно для степенных, просто необходимо знаменатель “превращать” в числитель, а корни — в степени, потому что в этом случае мы сможем воспользоваться самой простой и самой запоминающейся табличной формулой (x α) = αx α − 1 .

    Пример 4.

    Пример 5.


    В этих двух примерах, представлены обычные ошибки при дифференцировании дроби с константой, а в следующем примере переход от корня к дробной степени нужен потому, что иначе часто забывают, что подобная функция не является табличной и должна дифференцироваться по правилу для сложной функции.

    Пример 6.

  • Не надо путать слагаемые и сомножители (сумму и произведение).
  • Константа-слагаемое при дифференцировании обнуляется, константа-сомножитель при дифференцировании сохраняется.

    Кроме того, почему-то для многих учеников производную функции y = x 2 + 0,1 вычислить легче, чем такую же производную вида (0,1 + х 2)

    . И для производной функции y = 0,1х 2 часто догадываются о существовании первого правила, а для (х 2 ·0,1) нет.
    Если Вы допускаете ошибки такого рода, то вспомните, что от перестановки мест слагаемых сумма не изменяется, и от перестановки сомножителей произведение не изменяется. Переставьте их так, как вам удобнее, и аккуратно примените первое или второе правила дифференцирования.

    Пример 7.


  • Не надо путать степенную x а и показательную a x функции.
  • В первом случае переменная находится в основании степени, читаем: “икс в степени а”. Во втором — переменная в показателе степени, читаем “а в степени икс”. Функции разные, формулы для вычисления производных разные. См. .

    Пример 8.

    Пример 9.

    Это пример для продвинутых. Задумайтесь о том, как бы Вы вычислили производную функции y = x x , в которой переменную поместили и в основание, и в показатель степени.
    Хорошо подумав, но не раньше, кликните по , чтобы раскрыть мой ответ.

    Это сложная функция, которая не относится напрямую ни к классу степенных, ни к классу показательных. Для вычисления производной в таких случаях часто требуется произвести предварительные преобразования. Например, здесь сначала выражение прологарифмировали, затем нашли производные обеих частей равенства по своим переменным и, наконец, составили уравнение для нахождения нужной производной по переменной

    х .

  • Не надо забывать о том, что производная сложной функции вычисляется “с продолжением” до получения табличной формулы.
  • Сложная функция, это функция зависящая не напрямую от заданной переменной, а от другой функции. Иными словами, её значение нельзя вычислить в одно действие. Например, функции y = sinx 2

    и y = sin 2 x являются сложными. Посмотрим, как вычисляются их значения, например при х = 2.

    Для функции y = sinx 2 нужно сначала возвести x в квадрат: 2 2 = 4, а затем вычислить значение синуса 4-ёх. Сделаем это с помощью калькулятора: sin4 = −0,75680249530792825… ≈ −0,76 (не забудьте, что аргументы тригонометрических функций считаются заданными в радианах).

    Для функции y = sin 2 x сначала определяем значение синуса 2-ух с помощью калькулятора: sin2 = 0,9092974268256816…, а затем возводим это значение в квадрат sin 2 2 = (0,9092974268256816…) 2 = 0,82682181043180595… ≈ 0,83.

    Таким образом, мы сначала вычисляем значение внутренней функции, а затем используем его как аргумент для внешней.


    Согласно пятому правилу дифференцирования, при определении производной нужно поступать наоборот – сначала вычислять производную внешней функции по её аргументу, а затем умножать её на производную внутренней.

    Как я уже упоминала, в этой операции ошибаются чаще всего. Ошибки могут быть самые разные, распространены следующие три.

    1-я ошибка) Можно просто не применить нужное правило, “не заметив”, что функция сложная.
    В следующем примере формулы дифференцирования степенной и тригонометрической функций использованы не последовательно, а одновременно, производная неверно вычислена в одно действие.

    Пример 10.

    2-я ошибка) Можно не разобраться, где внутренняя, а где внешняя функции.
    В следующем примере показатель степени стоит над x , т.е. над аргументом, поэтому степенная функция внутренняя, а синус внешняя. Ученик воспринял это иначе, решил, что синус в квадрате и допустил ошибку.

    Пример 11.

    Чтобы избавиться от ошибок такого рода, научиться анализировать сложную функцию, отделять внутреннюю от внешней, нужно просто смотреть в каком порядке Вы бы проводили вычисления, и дифференцирование проводить в обратном порядке. При этом можно расставлять отсутствующие скобки, а если всё равно испытываете трудности, то вводить дополнительные обозначения. Что касается степеней, то можно запомнить следующее – над каким обозначением стоит показатель степени, то и является её основанием (возводится в степень).

    Пример 12.


    Здесь в конце использована тригонометрическая формула для того, чтобы записать ответ в наиболее компактной форме.

    Пример 13.


    Здесь в конце переставлены сомножители также для того, чтобы записать ответ в более компактной и удобочитаемой форме.

    3-я ошибка) Правило используется не до конца
    Один раз учли, что функция сложная и хватит. А если функция вложена несколько раз? Например, корень квадратный из суммы двух логарифмов с разными основаниями, первый из которых зависит от sinx , а второй от cosx . Или арктангенс, зависящий от натурального логарифма, который, в свою очередь, зависит от х в квадрате.

    Пример 14.


    Пример 15.

  • Не надо стесняться ставить скобки.
  • Предыдущий пример демонстрирует выход из положения с помощью введения дополнительных обозначений. Но, на мой взгляд, это всё-таки не самый оптимальный способ для длинных вычислений. Лучший подход к дифференцированию сложной функции – скобки, которые можно дописывать явно или, по мере укрепления навыка, представлять себе мысленно.
    Расставляем скобки и постепенно снаружи внутрь раскрываем их. Содержимое очередной скобки является переменной, по которой производится дифференцирование по формуле f u ·(u ) . Производную f u находим по таблице производных, заменяя в формуле x на u . Если всё сделано правильно, то процесс закончится тем, что содержимое последней, самой внутренней скобки полностью совпадёт с одной из табличных формул для производных.

    Пример 16.


    PS: В примерах 11 и 14 допущены ошибки, не только упомянутые в комментариях к ним, но ещё по одной стандартной ошибке. Заметили какие?

    Есть вопросы? пожелания? замечания?
    Обращайтесь –

    Внимание, ©mathematichka . Прямое копирование материалов на других сайтах запрещено. Ставьте гиперссылку.

    Приведена формула производной суммы и разности функций. Приведено доказательство и подробно разобраны примеры применения этой формулы.

    Содержание

    Формула производной суммы (разности) функций

    Пусть и являются функциями от независимой переменной x . Пусть они дифференцируемы в некоторой области значений переменной x . Тогда, в этой области, производная от суммы (разности) этих функций равна сумме (разности) производных этих функций :
    (1) .

    Доказательство

    Поскольку функции и дифференцируемы при , то существуют следующие пределы, которые являются производными этих функций:
    ;
    .

    Рассмотрим функцию y от переменной x , которая является суммой функций и :
    .
    Применим определение производной.

    .

    Тем самым мы доказали, что производная от суммы функций равна сумме производных:
    .

    Тем же способом можно показать, что производная от разности функций равна разности производных:
    .
    Это можно показать и другим способом, применяя только что доказанное правило дифференцирования суммы и :
    .

    Эти два правила можно записать в виде одного уравнения:
    (1) .

    Следствие

    Выше мы рассмотрели правило нахождения производной от суммы двух функций. Это правило можно обобщить на сумму и разность от любого числа дифференцируемых функций.

    Производная от суммы (разности) любого конечного числа дифференцируемых функций равна сумме (разности) их производных . С учетом правила вынесения постоянной за знак производной , это правило можно записать так:
    .
    Или в развернутом виде:
    (2) .
    Здесь – постоянные;
    – дифференцируемые функции от переменной x .

    Доказательство следствия

    При n = 2 , применим правило (1) и правило вынесения постоянной за знак производной . Имеем:
    .
    При n = 3 применим формулу (1) для функций и :
    .

    Для произвольного числа n применим метод индукции. Пусть уравнение (2) выполняется для . Тода для имеем:

    .
    То есть из предположения, что уравнение (2) выполняется для следует, что уравнение (2) выполняется для . А поскольку уравнение (2) выполняется для , то оно выполняется для всех .
    Следствие доказано.

    Примеры

    Пример 1

    Найдите производную
    .

    Раскрываем скобки. Для этого применим формулу
    .
    Также используем свойства степенных функций .
    ;

    ;
    .

    Применяем формулу (2) для производной от суммы и разности функций.
    .

    Из таблицы производных находим:
    .
    Тогда
    ;
    ;
    .

    Окончательно имеем:
    .

    Пример 2

    Найти производную от функции от переменной x
    .

    Приведем корни к степенным функциям .
    .
    Применяем правило дифференцирования суммы и разности.
    .
    Применяем формулы из таблицы производных .
    ;
    ;
    ;
    ;
    ;
    .
    Подставляем:
    .
    Приводим дроби к общему знаменателю.
    .
    Здесь мы учли, что заданная функция определена при .
    .

    Вычисление производной – одна из самых важных операций в дифференциальном исчислении. Ниже приводится таблица нахождения производных простых функций. Более сложные правила дифференцирования смотрите в других уроках:
    • Таблица производных экспоненциальных и логарифмических функций
    Приведенные формулы используйте как справочные значения. Они помогут в решении дифференциальных уравнений и задач. На картинке, в таблице производных простых функций, приведена “шпаргалка” основных случаев нахождения производной в понятном для применения виде, рядом с ним даны пояснения для каждого случая.

    Производные простых функций

    1. Производная от числа равна нулю
    с´ = 0
    Пример:
    5´ = 0

    Пояснение :
    Производная показывает скорость изменения значения функции при изменении аргумента. Поскольку число никак не меняется ни при каких условиях – скорость его изменения всегда равна нулю.

    2. Производная переменной равна единице
    x´ = 1

    Пояснение :
    При каждом приращении аргумента (х) на единицу значение функции (результата вычислений) увеличивается на эту же самую величину. Таким образом, скорость изменения значения функции y = x точно равна скорости изменения значения аргумента.

    3. Производная переменной и множителя равна этому множителю
    сx´ = с
    Пример:
    (3x)´ = 3
    (2x)´ = 2
    Пояснение :
    В данном случае, при каждом изменении аргумента функции (х ) ее значение (y) растет в с раз. Таким образом, скорость изменения значения функции по отношению к скорости изменения аргумента точно равно величине с .

    Откуда следует, что
    (cx + b)” = c
    то есть дифференциал линейной функции y=kx+b равен угловому коэффициенту наклона прямой (k).


    4. Производная переменной по модулю равна частному этой переменной к ее модулю
    |x|” = x / |x| при условии, что х ≠ 0
    Пояснение :
    Поскольку производная переменной (см. формулу 2) равна единице, то производная модуля отличается лишь тем, что значение скорости изменения функции меняется на противоположное при пересечении точки начала координат (попробуйте нарисовать график функции y = |x| и убедитесь в этом сами. Именно такое значение и возвращает выражение x / |x| . Когда x 0 – единице. То есть при отрицательных значениях переменной х при каждом увеличении изменении аргумента значение функции уменьшается на точно такое же значение, а при положительных – наоборот, возрастает, но точно на такое же значение.

    5. Производная переменной в степени равна произведению числа этой степени и переменной в степени, уменьшенной на единицу
    (x c)”= cx c-1 , при условии, что x c и сx c-1 ,определены а с ≠ 0
    Пример:
    (x 2)” = 2x
    (x 3)” = 3x 2
    Для запоминания формулы :
    Снесите степень переменной “вниз” как множитель, а потом уменьшите саму степень на единицу. Например, для x 2 – двойка оказалась впереди икса, а потом уменьшенная степень (2-1=1) просто дала нам 2х. То же самое произошло для x 3 – тройку “спускаем вниз”, уменьшаем ее на единицу и вместо куба имеем квадрат, то есть 3x 2 . Немного “не научно”, но очень просто запомнить.

    6. Производная дроби 1/х
    (1/х)” = – 1 / x 2
    Пример:
    Поскольку дробь можно представить как возведение в отрицательную степень
    (1/x)” = (x -1)” , тогда можно применить формулу из правила 5 таблицы производных
    (x -1)” = -1x -2 = – 1 / х 2

    7. Производная дроби с переменной произвольной степени в знаменателе
    (1 / x c)” = – c / x c+1
    Пример:
    (1 / x 2)” = – 2 / x 3

    8. Производная корня (производная переменной под квадратным корнем)
    (√x)” = 1 / (2√x) или 1/2 х -1/2
    Пример:
    (√x)” = (х 1/2)” значит можно применить формулу из правила 5
    (х 1/2)” = 1/2 х -1/2 = 1 / (2√х)

    9. Производная переменной под корнем произвольной степени
    (n √x)” = 1 / (n n √x n-1)

    С правочные материалы по теме «производная». Базовый школьный уровень.
    Теоретические сведения для учеников, преподавателей и репетиторов по математике. В помощь к проведению занятий.

    Определение: производной функции в точке называется предел отношения приращения функции к приращению переменной, то есть

    Таблица производных основных математических функций:

    Правила вычисления производных

    Производная суммы двух любых выражений равна сумме производных этих выражений (производная суммы равна сумме производных)

    Производная разности двух любых выражений равна разности производных этих слагаемых (производная разности равна разности производных).

    Производная от произведения двух множителей равна произведению производной первого множителя на второй плюс произведение первого множителя на производную второго (сумма поочередно взятых производных от множителей).
    Комментарий репетитора по математике: когда я короткими фразами напоминаю ученику о правиле вычисления производной от произведения, я говорю так: производная первого множителя на второй плюс обмен штрихами!


    Производная от частного двух выражений равна частному разности поочередно взятых производных от множителей и квадрата знаменателя.

    Производная от произведения числа на функцию . Чтобы найти производную от произведения числа на буквенное выражение (на функцию) нужно умножить это число на производную этого буквенного выражения.

    Производная сложной функции:

    Для вычисления производной сложной функции необходимо найти производную внешней функции и умножить ее на производную внутренней функции.

    Ваши комментарии и отзывы к странице с производными:
    Александр С.
    Очень нужна была таблица. В интернете одна из самых. За пояснения и правила тоже огромное спасибо. Хотя бы по одному примеру ещё к ним и вообще было бы отлично было. Еще раз огромное спасибо.

    Колпаков А.Н, репетитор по математике: хорошо, постараюсь в ближайшее время дополнить страницу примерами.

    Виртуальный математический справочник.
    Колпаков Александр Николаевич, репетитор по математике.

    алгоритм и примеры решений. Производная, правила и формулы дифференцирования

    Таблица производных элементарных функций

    Определение 1

    Вычисление производной называют дифференцированием . 2}$.

    Пусть функция y = f(x) определена в промежутке X. Производной функции y = f(x) в точке х o называется предел

    = .

    Если этот предел конечный, то функция f(x) называется дифференцируемой в точке x o ; при этом она оказывается обязательно и непрерывной в этой точке.

    Если же рассматриваемый предел равен  (или – ), то при условии, что функция в точке х o непрерывна, будем говорить, что функция f(x) имеет в точке х o бесконечную производную .

    Производная обозначается символами

    y , f (x o), , .

    Нахождение производной называется дифференцированием функции. Геометрический смысл производной состоит в том,что производная есть угловой коэффициент касательной к кривой y=f(x) в данной точке х o ; физический смысл – в том, что производная от пути по времени есть мгновенная скорость движущейся точки при прямолинейном движении s = s(t) в момент t o .

    Если с – постоянное число, и u = u(x), v = v(x) – некоторые дифференцируемые функции, то справедливы следующие правила дифференцирования:

    1) (с) ” = 0, (cu) ” = cu”;

    2) (u+v)” = u”+v”;

    3) (uv)” = u”v+v”u;

    4) (u/v)” = (u”v-v”u)/v 2;

    5) если y = f(u), u = (x), т.е. y = f((x)) – сложная функция, или суперпозиция , составленная из дифференцируемых функций  и f, то , или

    6) если для функции y = f(x) существует обратная дифференцируемая функция x = g(y), причем  0, то .

    На основе определения производной и правил дифференцирования можно составить список табличных производных основных элементарных функций.

    1. (u )” =  u  1 u” (  R ).

    2. (a u)” = a u lna u”.

    3. (e u)” = e u u”.

    4. (log a u)” = u”/(u ln a).

    5. (ln u)” = u”/u.

    6. (sin u)” = cos u u”.

    7. (cos u)” = – sin u u”.

    8. (tg u)” = 1/ cos 2 u u”.

    9. (ctg u)” = – u” / sin 2 u.

    10. (arcsin u)” = u” / .

    11. (arccos u)” = – u” / .

    12. (arctg u)” = u”/(1 + u 2).

    13. (arcctg u)” = – u”/(1 + u 2).

    Вычислим производную степенно-показательного выражения y=u v , (u>0), где u и v суть функции от х , имеющие в данной точке производные u” , v” .

    Прологарифмировав равенство y=u v , получим ln y = v ln u.

    Приравнивая производные по х от обеих частей полученного равенства с помощью правил 3, 5 и формулы для производной логарифмической функции, будем иметь:

    y”/y = vu”/u +v” ln u, откуда y” = y (vu”/u +v” ln u).

    (u v)”=u v (vu”/u+v” ln u), u > 0.

    Например, если y = x sin x , то y” = x sin x (sin x/x + cos x ln x).

    Если функция y = f(x) дифференцируема в точке x , т.е. имеет в этой точке конечную производную y” , то = y”+, где 0 при х 0; отсюда  y = y” х +  x.

    Главная часть приращения функции, линейная относительно х, называется дифференциалом функции и обозначается dy: dy = y” х. Если положить в этой формуле y=x, то получим dx = x”х = 1х =х, поэтому dy=y”dx, т. е. символ для обозначения производной можно рассматривать как дробь.

    Приращение функции  y есть приращение ординаты кривой, а дифференциал dy есть приращение ординаты касательной.

    Пусть мы нашли для функции y=f(x) ее производную y = f (x). Производная от этой производной называется производной второго порядка функции f(x), или второй производной, и обозначается .

    Аналогично определяются и обозначаются:

    производная третьего порядка – ,

    производная четвертого порядка –

    и вообще производная n-го порядка – .

    Пример 3 .15. Вычислить производную функции y=(3x 3 -2x+1)sin x.

    Решение. По правилу 3, y”=(3x 3 -2x+1)”sin x + (3x 3 -2x+1)(sin x)” = = (9x 2 -2)sin x + (3x 3 -2x+1)cos x.

    Пример 3.16 . Найти y”, y = tg x + .

    Решение. Используя правила дифференцирования суммы и частного, получим: y”=(tgx + )” = (tgx)” + ()” = + = .

    Пример 3 .17. Найти производную сложной функции y= , u=x 4 +1.

    Решение. По правилу дифференцирования сложной функции, получим: y” x =y ” u u” x =()” u (x 4 +1)” x =(2u + . Так как u=x 4 +1,то (2 x 4 +2+ .

    Дифференцирование – это вычисление производной.

    1. Формулы дифференцирования.

    Основные формулы дифференцирования – в таблице. Их необязательно зазубривать. Поняв некоторые закономерности, вы сможете из одних формул самостоятельно выводить другие.

    1) Начнем с формулы (kx + m)′ = k.
    Ее частными случаями являются формулы x ′ = 1 и C′ = 0.

    В любой функции вида у = kx + m производная равна угловому коэффициенту k.

    Например, дана функция у = 2х + 4. Ее производная в любой точке будет равна 2:

    (2 х + 4)′ = 2 .

    Производная функции у = 9 х + 5 в любой точке равна 9 . И т.д.

    А давайте найдем производную функции у = 5х . Для этого представим 5х в виде (5х + 0). Мы получили выражение, похожее на предыдущее. Значит:

    (5х )′ = (5х + 0)′ = 5.

    Наконец, выясним, чему равна x ′.
    Применим прием из предыдущего примера: представим х в виде 1х + 0. Тогда получим:

    x ′ = (1х + 0)′ = 1.

    Таким образом, мы самостоятельно вывели формулу из таблицы:

    (0 · x + m)′ = 0.

    Но тогда получается, что m′ тоже равна 0. Пусть m = C, где C – произвольная постоянная. Тогда мы приходим к еще одной истине: производная постоянной равна нулю. То есть получаем еще одну формулу из таблицы.

    При решении задач дифференцирования приходится искать производные функций различных классов. В этой статье мы рассмотрим основные правила дифференцирования , которые будем постоянно использовать при нахождении производных. Все эти правила докажем на основе определения производной функции и обязательно остановимся на подробном решении примеров, чтобы понять принцип их применения.

    При доказательстве правил дифференцирования будем считать функции f(x) и g(x) дифференцируемыми на некотором промежутке X .

    То есть, для любого справедливо , где – приращения соответствующих функций.

    В другой записи .

    К основным правилам дифференцирования относят:

    Вынесение постоянного множителя за знак производной.

    Докажем формулу . По определению производной имеем:

    Произвольный множитель можно выносить за знак предельного перехода (это известно из свойств предела), поэтому

    На этом доказательство первого правила дифференцирования завершено.

    Достаточно часто приходится сначала упрощать вид дифференцируемой функции, чтобы воспользоваться таблицей производных и правилами нахождения производных. Следующие примеры это наглядно подтверждают.

    Пример.

    Выполнить дифференцирование функции .

    Решение.

    По свойствам логарифмической функции можно перейти к записи . Осталось вспомнить производную логарифмической функции и вынести постоянный множитель:

    Пример.

    Решение.

    Преобразуем исходную функцию .

    Применяем правило вынесения множителя за знак производной и из таблицы берем производную показательной функции:

    Производная суммы, производная разности.

    Для доказательства второго правила дифференцирования воспользуемся определением производной и свойством предела непрерывной функции.

    Подобным образом можно доказать, что производная суммы (разности) n функций равна сумме (разности) n производных .

    Пример.

    Найти производную функции .

    Решение.

    Упростим вид исходной функции .

    Используем правило производной суммы (разности):

    В предыдущем пункте мы доказали, что постоянный множитель можно выносить за знак производной, поэтому

    Осталось воспользоваться таблицей производных:

    Производная произведения функций.

    Докажем правило дифференцирования произведения двух функций .

    Запишем предел отношения приращения произведения функций к приращению аргумента. Будем учитывать, что и (приращение функции стремиться к нулю при приращении аргумента, стремящемся к нулю).

    Что и требовалось доказать.

    Пример.

    Продифференцировать функцию .

    Решение.

    В данном примере . Применяем правило производной произведения:

    Обращаемся к таблице производных основных элементарных функций и получаем ответ:

    Пример.

    Найти производную функции .

    Решение.

    В этом примере . Следовательно,

    Давайте рассмотрим случай нахождения производной произведения трех функций. В принципе, по этой же системе можно дифференцировать произведение и четырех, и пяти, и двадцати пяти функций.

    Пример.

    Выполнить дифференцирование функции .

    Решение.

    Будем исходить из правила дифференцирования произведения двух функций. В качестве функции f(x) будем считать произведение (1+x)sinx , а в качестве g(x) возьмем lnx :

    Для нахождения вновь применяем правило производной произведения:

    Используем правило производной суммы и таблицу производных:

    Подставляем полученный результат:

    Как видите, порой приходится применять несколько правил дифференцирования в одном примере. Сложного в этом ничего нет, главное действовать последовательно и не мешать все в кучу.

    Пример.

    Найти производную функции .

    Решение.

    Функция представляет собой разность выражений и , поэтому

    В первом выражении выносим двойку за знак производной, а ко второму выражению применяем правило дифференцирования произведения:

    Производная частного двух функций (производная дроби).

    Докажем правило дифференцирования частного двух функций (дроби) . Стоит оговориться, что g(x) не обращается в ноль ни при каких x из промежутка X .

    Таблица производных элементарных функций

    Определение 1

    Вычисление производной называют дифференцированием .2}$.

    Урок 3: Вычисление производной – 100urokov.ru

    План урока:

    Производные некоторых элементарных функций

    Основные правила дифференцирования

    Производная сложной функции

     

    Производные некоторых элементарных функций

    Ранее мы для вычисления производных использовали ее определение. То есть каждый раз мы давали функции некоторое приращение ∆х, потом находили соответствующую ему величину ∆у, далее составляли отношение ∆у/∆х, после чего находили предел этого отношения при ∆х →0. Выполнение такого алгоритма довольно трудоемко. Поэтому на практике используются специальные формулы для вычисления производных.

    Нам известно несколько основных функций, которые в математике чаще называют элементарными. Например, элементарными являются линейная функция, степенная, показательная, логарифмическая. Также существует несколько различных тригонометрических функций (синус, косинус, тангенс), которые тоже считаются элементарными. Попытаемся вычислить для них производные.

    Начнем с линейной функции. В общем случае она выглядит так:

    где k и b – некоторые постоянные числа.

    Выберем произвольную точку х0 и дадим ей приращение ∆х, в результате чего мы придем в новую точку (х0 + ∆х). Вычислим значения линейной функции в этих двух точках:

    Теперь мы можем найти приращение функции ∆у:

    Находим отношение ∆у/∆х:

    Получилось, что это отношение не зависит ни от приращения ∆х, ни от выбора исходной точки х0. Естественно, что предел этого отношения при ∆х→0 (то есть производная) также будет равен k:

     

    Задание. Вычислите производную функции у = 4х + 9.

     

    Обратите внимание, что в рассмотренном примере запись у′ = 4 означает функцию. Просто при любом значении х она принимает одно и то же значение, равное 4. График производной функции будет выглядеть так:

    Рассмотрим два особых частных случая линейной функции. Пусть k = 1 и b = 0, тогда она примет вид у = х. Её производная тогда будет равна 1:

    Теперь предположим, что коэффициент k = 0. Тогда функция примет вид

    где С – некоторое постоянное число, то есть константа (большая буква Св таких случаях используется из-за латинского термина constanta). Производная такой функции будет равна нулю:

     

    Задание. Найдите вторую производную функции у = 9х + 2.

    Решение. Сначала вычислим первую производную:

    Очень легко объяснить, почему производная константы равна нулю. Представим себе, что закон движения некоторого тела выглядит как s(t) = C, например, s(t) = 5. Это значит, что тело в любой момент времени находится в точке, находящейся в 5 метрах от какого-то начала отсчета. То есть тело находится в одной и той же точке, а это значит, что оно не двигается. Тогда его скорость равна нулю. Но производная – это и есть скорость, значит, она также равна нулю.

    Далее вычислим производную для функции у = 1/х. Выберем некоторую точку х0 и дадим ей приращение ∆х. В результате имеем две точки с координатами х0 и (х0 + ∆х). Вычислим значение функции в каждой из них:

    Осталось найти предел данного отношения при ∆х→0. Ясно, что при этом множитель х0 + ∆х будет стремится к х0, то есть

     

    Задание. Вычислите производные функции

     

    Обратите внимание, что производная функции у = 1/х оказывается отрицательной при любом значении х (кроме нуля, для которого производную посчитать нельзя, так как получится деление на ноль). Это должно означать, что функция убывает в каждой своей точке, а любая касательная к ней образует с осью Ох тупой угол наклона. И это действительно так:

    Мы разобрали несколько простейших примеров того, как находить формулы производных. Для этого используется понятие предела функции. Для вывода всех подобных формул требуется хорошо знать тему вычисления пределов, которая не изучается детально в школе. Поэтому мы просто дадим следующие формулы без доказательств.

    Начнем со степенной функции у = хn, где n– некоторое постоянное число. Её производная вычисляется по формуле:

    Приведем примеры использования этой формулы:

    Задание. Найдите производную функции у = х6 в точке х0 = 10.

     

    Задание. Движение самолета при разгоне описывается законом движения s(t) = t3. Найдите его скорость через 5 секунд после начала разгона.

    Решение. Скорость самолета в любой момент времени равна производнойs′(t). Найдем её:

    Заметим, что используемая нами формула работает и в том случае, если показатель степени является отрицательным или дробным числом. Действительно, ранее мы вывели формулу

    По определению отрицательной степени мы можем записать, что

     

    Задание. Вычислите производную функции

     

    Задание. Определите, в какой точке необходимо провести касательную к графику функции

    чтобы она образовывала с осью Ох угол в 45°?

    Решение. Тангенс угла наклона касательной равен производной. Известно, что tg 45° = 1. Значит, нам надо найти такую точку х0, в которой значение производной квадратного корня будет равно единице. Производная вычисляется по формуле:

    Ответ: х0 = 0,25.

    Далее изучим формулы производных для тригонометрических функций. Они выглядят так:

    Рассмотрим несколько примеров использования этих формул.

     

    Задание. Найдите производную функции у = cosx в точке х0 = π.

    Решение. Мы знаем, что

     

    Задание. Найдите угол наклона касательной, проведенной к графику у = sinx в начале координат.

    Решение. Производная синуса вычисляется по формуле:

    Получается, что тангенс угла наклона также равен единице. Это значит, что сам угол равен 45°. Построение показывает, что это действительно так:

     

    Задание. Найдите производную функции у = tgx в точке х0 = π/6.

    Решение. Для тангенса используется формула:

    Далее рассмотрим показательную и логарифмическую функцию. Их производные рассчитываются по следующим формулам:

    Обратите внимание, что в этих формулах появился натуральный логарифм, то есть логарифм, основанием которого является число е. Именно из-за наличия натурального логарифма в формулах дифференцирования он играет особо важную роль в математике и имеет отдельное обозначение. Вычислим несколько производных с помощью приведенных формул:

    Напомним, что справедлива формула

    Стоит обратить внимание, что функции у = ех при дифференцировании не меняется. Эта особенность функции также имеет огромное значение в математическом анализе.

     

    Задание. Найдите угол наклона касательных, проведенных к графику у = ех в точке (0; 1) и к графику у = lnx в точке (1; 0).

    Решение. Используем формулы производных:

    Получили, что тангенс наклона касательной равен 1. Из этого следует, что угол наклона касательной равен 45°. Далее найдем производную натурального логарифма при х = 1:

    Производная снова равна 1, значит, угол наклона также составит 45°, что подтверждается рисунком:

    Ответ: 45°.

     

    Задание. Вычислите производную функции у = 2х при х0 = 3.

    Решение. Используем формулу

    Сведем использованные нами равенства в одну таблицу производных основных функций:

    Основные правила дифференцирования

    До этого мы рассматривали довольно простые, то есть стандартные функции, для каждой из которых производную можно узнать из справочника или таблицы. Но что делать, если нам потребовалось продифференцировать функцию, которая состоит из нескольких основных? Например, что делать с функциями у = 5х2 + 6х – 3 или у = x•sinx?

    Все более сложные функции можно получить из нескольких простых, комбинируя их. Так, функция у = х3 + х2 получается сложением функций у = х3 и у = х2, а функция у = (lnx)•(cosx) – произведением функций у = lnx и у = сosx.

    Есть несколько правил, которые позволяют находить производные в таких случаях. Мы не будем их доказывать, а просто дадим их формулировки. Также будем нумеровать правила. Первое из них помогает находить производную сумму функций.

    В данном случае u и v – это просто обозначение каких-то произвольных функций. Рассмотрим пример. Пусть надо найти производную функции

    Правило работает и в том случае, если сумма представляет собой сумму не двух, а большего числа слагаемых:

    Следующее правило позволяет выносить постоянный множитель за знак производной:

    Покажем использование этого правила:

    Действительно, зная эти формулы и первые два правила вычисления производных, мы можем записать, что

     

    Задание. Вычислите значение производной функции у = 9х3 + 7х2 – 25х + 7 в точке х0 = 1.

    Решение. Пользуясь правилами дифференцирования, находим производную:

    Несколько сложнее обстоит дело с дифференцированием функций, получающихся при перемножении простых функций. В таких случаях используется следующее правило:

    Предположим, надо найти производную для функции у = х2•sinx. Её можно представить как произведение u•v, где

    Примечание. В последнем случае мы в конце примера использовали формулу косинуса двойного угла:

    Заметим, что иногда одно и то же задание с производной можно решить по-разному, используя или не используя правило для вычисления производной произведения функций.

     

    Задание. Найдите производную функции у = х2•(3х + х3). Вычислите ее значение при х = 1.

    Решение. Функция у представляет собой произведение более простых функций u•v, где

     

    Задание. Продифференцируйте функцию

    Решение. Здесь перед нами функция, которая представляет собой произведение сразу трех множителей. Что делать в таком случае? Надо всего лишь добавить скобки и их помощью оставить только два множителя (один их них окажется «сложным»):

    Довольно сложно выглядит формула для поиска производной дроби:

    Например, пусть надо найти производную функции

    С помощью данного правила можно доказать некоторые равенства. Так, ранее мы уже записали (без доказательства) формулы производных тригонометрических функций:

    Оказывается, формула для тангенса может быть выведена из формул для синуса и косинуса. Действительно, тангенс можно записать в виде дроби:

     

    Задание. Найдите, в каких точках надо провести касательную к графику дробно-линейной функции

    чтобы эта касательная образовала с осью Ох угол в 135°.

    Решение. Угол будет равен 135° только тогда, когда значение производной будет равно (– 1) (так как tg 135° = – 1). Поэтому сначала найдем производную. В данном случае следует использовать правило 4, так как функция у явно записана как дробь:

    Получили два значения х. Построив график и проведя касательные, мы убедимся, что они действительно образуют с осью Ох угол 135°:

    Ответ: – 2 и 0.

    Заметим, что иногда можно избавиться от необходимости использовать правило 4, если дифференцируемую функцию можно преобразовать. При этом часто помогает использование отрицательных степеней. Пусть надо продифференцировать функцию

    Напрашивается решение использовать правило 4.И такой путь позволит получить правильное решение, хотя и будет несколько трудоемким. Однако можно преобразовать функцию:

    У нас получилось произведение, а потому можно использовать правило 3, которое представляется более простым:

    Производная сложной функции

    «Сконструировать» громоздкую функцию из нескольких простых можно не только с помощью арифметических действий. Например, возьмем функции

    В обоих случаях мы получили некоторую функцию, продифференцировать которую с помощью уже известных нам правил не получится. Функции, сконструированные таким образом, называются сложными. Есть универсальная формула, позволяющая находить производную сложной функции:

    Посмотрим, как пользоваться эти правилом. Пусть надо вычислить производную функции

    Она сконструирована из функции у = ex и у = sinx, причем вторая подставлена в первую. Это значит, что первую можно обозначить буквой u, а вторую – буквой v (если использовать обозначения в правиле 5):

    Задание. Найдите у′, если у = sin 2x.

    Решение. На этот раз в качестве исходной функции выступает

    Убедиться в справедливости правила 5 можно на примере функции

    Её можно продифференцировать двумя разными способами. Сначала попробуем просто избавиться от квадрата в исходной функции, используя формулу квадрата суммы:

    В результате оба способа вычисления производной дали одинаковый ответ.

     

    Задание. Найдите производную сложной функции у = (2х + 5)1000.

    Решение. В данном случае мы рассматриваем комбинацию следующих функций:

    Теперь мы умеем вычислять производные почти любых функций, которые можно записать с помощью элементарных функций и арифметических операций. При этом нам не надо использовать определение понятия производной и вычислять какие бы то ни было пределы. Достаточно знать производные основных функций и несколько (всего лишь 5) правил дифференцирования. Навыки дифференцирования функций пригодятся в будущем при решении практических задач, связанных с производными.

     

    Производная. Таблица производных. Связь функции с производной. Касательная. Первообразная

    Факт 1.
    Таблица производных: \[\begin{array}{|r|c|c|} \hline & \text{Функция } f(x) & \text{Производная } f'(x)\\ \hline \textbf{1} & c & 0\\&&\\ \textbf{2} & x^a & a\cdot x^{a-1}\\&&\\ \textbf{3} & \ln x & \dfrac1x\\&&\\ \textbf{4} & \log_ax & \dfrac1{x\cdot \ln a}\\&&\\ \textbf{5} & e^x & e^x\\&&\\ \textbf{6} & a^x & a^x\cdot \ln a\\&&\\ \textbf{7} & \sin x & \cos x\\&&\\ \textbf{8} & \cos x & -\sin x\\ \hline \end{array} \quad \quad \quad \quad \begin{array}{|r|c|c|} \hline & \text{Функция } f(x) & \text{Производная } f'(x)\\ \hline &&\\ \textbf{9} & \mathrm{tg}\, x & \dfrac1{\cos^2 x}\\&&\\ \textbf{10} & \mathrm{ctg}\, x & -\,\dfrac1{\sin^2 x}\\&&\\ \textbf{11} & \arcsin x & \dfrac1{\sqrt{1-x^2}}\\&&\\ \textbf{12} & \arccos x & -\,\dfrac1{\sqrt{1-x^2}}\\&&\\ \textbf{13} & \mathrm{arctg}\, x & \dfrac1{1+x^2}\\&&\\ \textbf{14} & \mathrm{arcctg}\, x & -\,\dfrac1{1+x^2}\\[4ex] \hline \end{array}\]

     

    Факт 2.2}\] \(\bullet\) Производная сложной функции: \[\big(h(f(x))\big)’=h’_f(f)\cdot f’_x(x)\]

     

    Факт 3.
    \(\bullet\) Если \(y=f(x)\) – некоторая функция, то касательная к ней в точке с абсциссой \(x_0\) имеет вид: \[y=f(x_0)+f'(x_0)\cdot (x-x_0)\] \(\bullet\) Следовательно, \(k=f'(x_0)=\mathrm{tg}\,\alpha\) – тангенс угла наклона касательной к положительному направлению оси \(Ox\), он же угловой коэффициент касательной, если ее уравнение записать как \(y=kx+b\).


     

    Факт 4.
    \(\bullet\) Если \(f'(x)>0\) на \((a;b)\), то \(f(x)\) возрастает на \((a;b)\).
    \(\bullet\) Если \(f'(x)<0\) на \((a;b)\), то \(f(x)\) убывает на \((a;b)\).
    \(\bullet\) Если \(f'(x_0)=0\) и в точке \(x_0\) производная меняет свой знак, то \(x_0\) — функции \(f(x)\):
    — если производная меняет знак с “\(-\)” на “\(+\)” (считая слева направо), то \(x_0\) — ;
    — если производная меняет знак с “\(+\)” на “\(-\)” (считая слева направо), то \(x_0\) — .b f(x)\,dx=F(b)-F(a)\] \(\bullet\) Тогда \(F(b)-F(a)\) равно площади закрашенной фигуры \(ABCD\), называемой криволинейной трапецией:


     

    Как найти произведение функции

    Данная функция является сложной, потому производную будем брать по цепочке. Сначала от внешней функции, затем от внутренней. При этом выполняя их перемножение.

    $$ y’ = (ln sin 3x )’ = frac<1> <sin 3x>cdot (sin 3x)’ = $$

    Заметим, что аргумент синуса отличен от $ x $, поэтому тоже является сложной функцией:

    $$ = frac<1> <sin 3x>cdot cos 3x cdot (3x)’ = frac<1> <sin 3x>cdot cos 3x cdot 3 $$

    Учитывая определение котангенса $ ctg x = frac<cos 3x> <sin 3x>$ перепишем полученную производную в удобном компактном виде:

    Как найти производную, как взять производную? На данном уроке мы научимся находить производные функций. Но перед изучением данной страницы я настоятельно рекомендую ознакомиться с методическим материалом Горячие формулы школьного курса математики. Справочное пособие можно открыть или закачать на странице Математические формулы и таблицы. Также оттуда нам потребуется Таблица производных, ее лучше распечатать, к ней часто придется обращаться, причем, не только сейчас, но и в оффлайне.

    Есть? Приступим. У меня для Вас есть две новости: хорошая и очень хорошая. Хорошая новость состоит в следующем: чтобы научиться находить производные, совсем не обязательно знать и понимать, что такое производная. Более того, определение производной функции, математический, физический, геометрический смысл производной целесообразнее переварить позже, поскольку качественная проработка теории, по моему мнению, требует изучения ряда других тем, а также некоторого практического опыта.
    И сейчас наша задача освоить эти самые производные технически. Очень хорошая новость состоит в том, что научиться брать производные не так сложно, существует довольно чёткий алгоритм решения (и объяснения) этого задания, интегралы или пределы, например, освоить труднее.

    Советую следующий порядок изучения темы: во-первых, эта статья. Затем нужно прочитать важнейший урок Производная сложной функции. Эти два базовых занятия позволят поднять Ваши навыки с полного нуля. Далее можно будет ознакомиться с более сложными производными в статье Сложные производные. Логарифмическая производная. Если планка окажется слишком высока, то сначала прочитайте вещь Простейшие типовые задачи с производной. Помимо нового материала, на уроке рассмотрены другие, более простые типы производных, и есть прекрасная возможность улучшить свою технику дифференцирования. Кроме того, в контрольных работах почти всегда встречаются задания на нахождение производных функций, которые заданы неявно или параметрически. Такой урок тоже есть: Производные неявных и параметрически заданных функций.

    Я попытаюсь в доступной форме, шаг за шагом, научить Вас находить производные функций. Вся информация изложена подробно, простыми словами.

    Собственно, сразу рассмотрим пример:

    Найти производную функции

    Решение:

    Это простейший пример, пожалуйста, найдите его в таблице производных элементарных функций. Теперь посмотрим на решение и проанализируем, что же произошло? А произошла следующая вещь: у нас была функция , которая в результате решения превратилась в функцию .

    Говоря совсем просто, для того чтобы найти производную функции, нужно по определенным правилам превратить её в другую функцию. Посмотрите еще раз на таблицу производных – там функции превращаются в другие функции. Единственным исключением является экспоненциальная функция , которая превращается сама в себя. Операция нахождения производной называется дифференцированием.

    Обозначения: Производную обозначают или .

    ВНИМАНИЕ, ВАЖНО! Забыть поставить штрих (там, где надо), либо нарисовать лишний штрих (там, где не надо) – ГРУБАЯ ОШИБКА! Функция и её производная – это две разные функции!

    Вернемся к нашей таблице производных. Из данной таблицы желательно запомнить наизусть: правила дифференцирования и производные некоторых элементарных функций, особенно:

    производную константы:
    , где – постоянное число;

    производную степенной функции:
    , в частности: , , .

    Зачем запоминать? Данные знания являются элементарными знаниями о производных. И если Вы не сможете ответить преподавателю на вопрос «Чему равна производная числа?», то учеба в ВУЗе может для Вас закончиться (лично знаком с двумя реальными случаями из жизни). Кроме того, это наиболее распространенные формулы, которыми приходится пользоваться практически каждый раз, когда мы сталкиваемся с производными.

    В реальности простые табличные примеры – редкость, обычно при нахождении производных сначала используются правила дифференцирования, а затем – таблица производных элементарных функций.

    В этой связи переходим к рассмотрению правил дифференцирования:

    1) Постоянное число можно (и нужно) вынести за знак производной

    , где – постоянное число (константа)

    Найти производную функции

    Смотрим в таблицу производных. Производная косинуса там есть, но у нас .

    Самое время использовать правило, выносим постоянный множитель за знак производной:

    А теперь превращаем наш косинус по таблице:

    Ну и результат желательно немного «причесать» – ставим минус на первое место, заодно избавляясь от скобок:

    2) Производная суммы равна сумме производных

    Найти производную функции

    Решаем. Как Вы, наверное, уже заметили, первое действие, которое всегда выполняется при нахождении производной, состоит в том, что мы заключаем в скобки всё выражение и ставим штрих справа вверху:

    Применяем второе правило:

    Обратите внимание, что для дифференцирования все корни, степени нужно представить в виде , а если они находятся в знаменателе, то переместить их вверх. Как это сделать – рассмотрено в моих методических материалах.

    Теперь вспоминаем о первом правиле дифференцирования – постоянные множители (числа) выносим за знак производной:

    Обычно в ходе решения эти два правила применяют одновременно (чтобы не переписывать лишний раз длинное выражение).

    Все функции, находящиеся под штрихами, являются элементарными табличными функциями, с помощью таблицы осуществляем превращение:

    Можно всё оставить в таком виде, так как штрихов больше нет, и производная найдена. Тем не менее, подобные выражения обычно упрощают:

    Все степени вида желательно снова представить в виде корней, степени с отрицательными показателями – сбросить в знаменатель. Хотя этого можно и не делать, ошибкой не будет.

    Найти производную функции

    Попробуйте решить данный пример самостоятельно (ответ в конце урока). Желающие также могут воспользоваться интенсивным курсом в pdf-формате, который особенно актуален, если у вас в распоряжении совсем мало времени.

    3) Производная произведения функций

    Вроде бы по аналогии напрашивается формула …., но неожиданность состоит в том, что:

    Эта необычное правило (как, собственно, и другие) следует из определения производной. Но с теорией мы пока повременим – сейчас важнее научиться решать:

    Найти производную функции

    Здесь у нас произведение двух функций, зависящих от .
    Сначала применяем наше странное правило, а затем превращаем функции по таблице производных:

    Сложно? Вовсе нет, вполне доступно даже для чайника.

    Найти производную функции

    В данной функции содержится сумма и произведение двух функций – квадратного трехчлена и логарифма . Со школы мы помним, что умножение и деление имеют приоритет перед сложением и вычитанием.

    Здесь всё так же. СНАЧАЛА мы используем правило дифференцирования произведения:

    Теперь для скобки используем два первых правила:

    В результате применения правил дифференцирования под штрихами у нас остались только элементарные функции, по таблице производных превращаем их в другие функции:


    Готово.

    При определенном опыте нахождения производных, простые производные вроде не обязательно расписывать так подробно. Вообще, они обычно решаются устно, и сразу записывается, что .

    Найти производную функции

    Это пример для самостоятельного решения (ответ в конце урока)

    4) Производная частного функций

    В потолке открылся люк, не пугайся, это глюк.
    А вот это вот суровая действительность:

    Найти производную функции

    Чего здесь только нет – сумма, разность, произведение, дробь…. С чего бы начать?! Есть сомнения, нет сомнений, но, В ЛЮБОМ СЛУЧАЕ для начала рисуем скобочки и справа вверху ставим штрих:

    Теперь смотрим на выражение в скобках, как бы его упростить? В данном случае замечаем множитель, который согласно первому правилу целесообразно вынести за знак производной:

    Заодно избавляемся от скобок в числителе, которые теперь не нужны.
    Вообще говоря, постоянные множители при нахождении производной можно и не выносить, но в этом случае они будут «путаться под ногами», что загромождает и затрудняет решение.

    Смотрим на наше выражение в скобках. У нас есть сложение, вычитание и деление. Со школы мы помним, что деление выполняется в первую очередь. И здесь – сначала применяем правило дифференцирования частного:

    Таким образом, наша страшная производная свелась к производным двух простых выражений. Применяем первое и второе правило, здесь это сделаем устно, надеюсь, Вы уже немного освоились в производных:

    Штрихов больше нет, задание выполнено.

    На практике обычно (но не всегда) ответ упрощают «школьными» методами:

    Найти производную функции

    Это пример для самостоятельного решения (ответ в конце урока).

    Время от времени встречаются хитрые задачки:

    Найти производную функции

    Смотрим на данную функцию. Здесь снова дробь. Однако перед тем как использовать правило дифференцирования частного (а его можно использовать), всегда имеет смысл посмотреть, а нельзя ли упростить саму дробь, или вообще избавиться от нее?
    Дело в том, что формула достаточно громоздка, и применять ее совсем не хочется.

    В данном случае можно почленно поделить числитель на знаменатель.
    Преобразуем функцию:

    Ну вот, совсем другое дело, теперь дифференцировать просто и приятно:

    Найти производную функции

    Здесь ситуация похожа, превратим нашу дробь в произведение, для этого поднимем экспоненту в числитель, сменив у показателя знак:

    Произведение все-таки дифференцировать проще:

    Найти производную функции

    Это пример для самостоятельного решения (ответ в конце урока).

    5) Производная сложной функции

    Данное правило также встречается очень часто. Но о нём рассказать можно очень много, поэтому я создал отдельный урок на тему Производная сложной функции.

    Пример 4: . В ходе решения данного примера следует обратить внимание, на тот факт, что и – постоянные числа, не важно чему они равны, важно, что это – константы. Поэтому выносится за знак производной, а .

    Пример 7:

    Пример 9:

    Пример 12:

    Автор: Емелин Александр

    (Переход на главную страницу)

    Профессиональная помощь по любому предмету – Zaochnik.com

    Как найти производную от дроби. Производная логарифмической функции

    Основные правила дифференцирования. Сумма.

    Выведем несколько правил вычисления производных, В этом пункте значения функций u и v и их производных в точке х 0 обозначаются для краткости так: u(х 0) = u, v(х 0) = v, u”(х 0) = u”, v”(х 0)=v`. Если функции u и v дифференцируемы в точке х 0 , то их сумма дифференцируема в этой точке и

    (u+v)” = u” + v” .

    Коротко говорят: производная суммы равна сумме производных . 1) Для доказательства вычислим сначала приращение суммы функций в рассматриваемой точке: Δ(u+v) = u (х 0 +Δx)+ v(х 0 +Δx) – (u(х 0)+v(х 0)) = (u(х 0 +Δx)-u(х 0)) + (v(х 0 +Δx)-v(х 0)) = Δu + Δv 2)

    3) Функции u и v дифференцируемы в точке х 0 , т. е. при Δх→0

    при Δх→0 (см. правило 3, а) предельного перехода ), т. е. (u+v)” = u”+v’

    Основные правила дифференцирования. Произведение.

    Если функции и и v дифференцируемы в точке х 0 , то их произведение дифференцируемо в этой точке и

    (uv)” = u”v+uv” .

    1) Найдем сначала приращение произведения:

    Δ(uv) = u(х 0 +Δx)v(х 0 +Δx)-u(х 0)v(х 0)=(u(х 0)+ Δu)(v(х 0)+ Δv)-u(х 0)v(х 0) =

    U(х 0)v(х 0)+ Δuv(х 0)+u(х 0) Δv+ΔuΔv-u(х 0)v(х 0)= Δuv(х 0)+u(х 0) Δv+ΔuΔv

    3) В силу дифференцируемости функций u и v в точке х 0 при Δx→0 имеем

    т. е. (uv)” = u”v+uv”, что и требовалось доказать. Следствие. Если функция u дифференцируема в х 0 , а С – постоянная, то функция Сu дифференцируема в этой точке и

    (Сu)” = Сu” .

    Коротко говорят: постоянный множитель можно выносить за знак производной . Для доказательства воспользуемся правилом 2 и известным из пункта о производной , фактом С” = 0:

    (Сu)” = Сu” + С”u = Cu” + 0⋅u = Cu”.

    Пример.

    Продифференцировать функцию .

    Решение.

    В данном примере . Применяем правило производной произведения:

    Обращаемся к таблице производных основных элементарных функций и получаем ответ:

    Основные правила дифференцирования. Частное

    Если функции u и v дифференцируемы в точке x 0 и функция v не равна нулю в этой точке, то частное u/v также дифференцируемо в x 0 и

    Выведем сначала формулу

    1) найдем приращение функции 1/v:

    2) Отсюда

    3) При Δx→0 имеем Δv/Δx→v’ (в силу дифференцируемости v в точке x 0), Δv→0 (по доказанной лемме ). Поэтому

    Теперь, пользуясь правилом нахождения производной произведения функций, находим производную частного:

    Пример.

    Выполнить дифференцирование функции .

    Решение.

    Исходная функция представляет собой отношение двух выражений sinx и 2x+1 . Применим правило дифференцирования дроби:

    Не обойтись без правил дифференцирования суммы и вынесения произвольной постоянной за знак производной:

    Производная сложной функции.

    Если функция f имеет производную в точке х 0 , а функция g имеет производную в точке y 0 =f(x 0 )y то сложная функция h(х) = g(f(х)) также имеет производную в точке х 0 , причем

    h’(x 0 ) = g’(f(x 0 )) f’(x 0 ) (1)

    Для доказательства формулы (1) надо (как и раньше) при Δx≠0 рассмотреть дробь Δh/Δx и установить, что

    при Δx→0. Введем обозначения:

    Δy = f(x 0 +Δx)-f(x 0)= Δf

    Тогда Δh = h(х 0 + Δх) – h(x 0) = g(f(x 0 +Δx)) – g(f(x 0)) = g(y 0 + Δy) – g(y 0) = Δg. Δy→0 при Δx→0, так как f дифференцируема в точке x 0 . Далее доказательство мы проведем только для таких функций f, у которых Δf≠0 в некоторой окрестности точки х 0 . Тогда

    при Δx→0, так как Δf/Δx→f’(x 0) при Δx→0, а Δg/Δy→g’(y 0) при Δy→0, что выполнено при Δx→0.

    Пример.НА ВСЯКИЙ СЛУЧАЙ!! ! ! !!! http://www.mathelp.spb.ru/book1/proizvodnaya.htm

    Производная обратной функции.

    Пусть функция дифференцируема и строго монотонна на . Пусть также в точке производная . Тогда в точке определена дифференцируемая функция , которую называют обратной к , а ее производная вычисляется по формуле .

    Найти производную обратной тригонометрической функции y = arcsinx. Обратная функция x = siny и , по формуле для обратной функции .

    Найдем функции y = arctgx. Обратная функция x = tgy,

    Производная суммы, производная разности.

    Для доказательства второго правила дифференцирования воспользуемся определением производной и свойством предела непрерывной функции.

    Подобным образом можно доказать, что производная суммы (разности) n функций равна сумме (разности) n производных

    Пример.

    Найти производную функции

    Решение.

    Упростим вид исходной функции

    Используем правило производной суммы (разности):

    В предыдущем пункте мы доказали, что постоянный множитель можно выносить за знак производной, поэтому

    Осталось воспользоваться таблицей производных:

    Докажем правило дифференцирования частного двух функций (дроби) . Стоит оговориться, что g(x) не обращается в ноль ни при каких x из промежутка X .

    По определению производной

    Пример.

    Выполнить дифференцирование функции .

    Решение.

    Исходная функция представляет собой отношение двух выражений sinx и 2x+1 . Применим правило дифференцирования дроби:

    Не обойтись без правил дифференцирования суммы и вынесения произвольной постоянной за знак производной:

    В заключении, давайте соберем все правила в одном примере.

    Пример.

    Найти производную функции , где a – положительное действительное число.

    Решение.

    А теперь по порядку.

    Первое слагаемое .

    Второе слагаемое

    Третье слагаемое

    Собираем все вместе:

    4.Вопрос.Производные Основных элементарных функций.

    Задание. Найти производную функции

    Решение. Используем правила дифференцирования и таблицу производных:

    Ответ.

    5.Вопрос.Производная сложной функции примеры

    Все примеры этого раздела опираются на таблицу производных и теорему о производной сложной функции, формулировка которой такова:

    Пусть 1) функция u=φ(x) имеет в некоторой точке x0 производную u′x=φ′(x0), 2) функция y=f(u) имеет в соответствующей точке u0=φ(x0) производную y′u=f′(u). Тогда сложная функция y=f(φ(x)) в упомянутой точке также будет иметь производную, равную произведению производных функций f(u) и φ(x):

    (f(φ(x)))′=f′u(φ(x0))⋅φ′(x0)

    или, в более короткой записи: y′x=y′u⋅u′x.

    В примерах этого раздела все функции имеют вид y=f(x) (т.е. рассматриваем лишь функции одной переменной x). Соответственно, во всех примерах производная y′ берётся по переменной x. Чтобы подчеркнуть то, что производная берётся по переменной x, часто вместо y′ пишут y′x.

    В примерах №1, №2 и №3 изложен подробный процесс нахождения производной сложных функций. Пример №4 предназначен для более полного понимания таблицы производных и с ним имеет смысл ознакомиться.

    Желательно после изучения материала в примерах №1-3 перейти к самостоятельному решению примеров №5, №6 и №7. Примеры №5, №6 и №7 содержат краткое решение, чтобы читатель мог проверить правильность своего результата.

    Пример №1

    Найти производную функции y=ecosx.

    Решение

    Нам нужно найти производную сложной функции y′. Так как y=ecosx, то y′=(ecosx)′. Чтобы найти производную (ecosx)′ используем формулу №6 из таблицы производных. Дабы использовать формулу №6 нужно учесть, что в нашем случае u=cosx. Дальнейшее решение состоит в банальной подстановке в формулу №6 выражения cosx вместо u:

    y′=(ecosx)′=ecosx⋅(cosx)′(1.1)

    Теперь нужно найти значение выражения (cosx)′. Вновь обращаемся к таблице производных, выбирая из неё формулу №10. Подставляя u=x в формулу №10, имеем: (cosx)′=−sinx⋅x′. Теперь продолжим равенство (1.1), дополнив его найденным результатом:

    y′=(ecosx)′=ecosx⋅(cosx)′=ecosx⋅(−sinx⋅x′)(1.2)

    Так как x′=1, то продолжим равенство (1.2):

    y′=(ecosx)′=ecosx⋅(cosx)′=ecosx⋅(−sinx⋅x′)=ecosx⋅(−sinx⋅1)=−sinx⋅ecosx(1.3)

    Итак, из равенства (1.3) имеем: y′=−sinx⋅ecosx. Естественно, что пояснения и промежуточные равенства обычно пропускают, записывая нахождение производной в одну строку, – как в равенстве (1.3). Итак, производная сложной функции найдена, осталось лишь записать ответ.

    Ответ : y′=−sinx⋅ecosx.

    Пример №2

    Найти производную функции y=9⋅arctg12(4⋅lnx).

    Решение

    Нам необходимо вычислить производную y′=(9⋅arctg12(4⋅lnx))′. Для начала отметим, что константу (т.е. число 9) можно вынести за знак производной:

    y′=(9⋅arctg12(4⋅lnx))′=9⋅(arctg12(4⋅lnx))′(2.1)

    Теперь обратимся к выражению (arctg12(4⋅lnx))′. Чтобы выбрать нужную формулу из таблицы производных было легче, я представлю рассматриваемое выражение в таком виде: ((arctg(4⋅lnx))12)′. Теперь видно, что необходимо использовать формулу №2, т.е. (uα)′=α⋅uα−1⋅u′. В эту формулу подставим u=arctg(4⋅lnx) и α=12:

    Дополняя равенство (2.1) полученным результатом, имеем:

    y′=(9⋅arctg12(4⋅lnx))′=9⋅(arctg12(4⋅lnx))′=108⋅(arctg(4⋅lnx))11⋅(arctg(4⋅lnx))′(2.2)

    Примечание: показать\скрыть

    Теперь нужно найти (arctg(4⋅lnx))′. Используем формулу №19 таблицы производных, подставив в неё u=4⋅lnx:

    (arctg(4⋅lnx))′=11+(4⋅lnx)2⋅(4⋅lnx)′

    Немного упростим полученное выражение, учитывая (4⋅lnx)2=42⋅(lnx)2=16⋅ln2x.

    (arctg(4⋅lnx))′=11+(4⋅lnx)2⋅(4⋅lnx)′=11+16⋅ln2x⋅(4⋅lnx)′

    Равенство (2.2) теперь станет таким:

    y′=(9⋅arctg12(4⋅lnx))′=9⋅(arctg12(4⋅lnx))′==108⋅(arctg(4⋅lnx))11⋅(arctg(4⋅lnx))′=108⋅(arctg(4⋅lnx))11⋅11+16⋅ln2x⋅(4⋅lnx)′(2.3)

    Осталось найти (4⋅lnx)′. Вынесем константу (т.е. 4) за знак производной: (4⋅lnx)′=4⋅(lnx)′. Для того, чтобы найти (lnx)′ используем формулу №8, подставив в нее u=x: (lnx)′=1x⋅x′. Так как x′=1, то (lnx)′=1x⋅x′=1x⋅1=1x. Подставив полученный результат в формулу (2.3), получим:

    y′=(9⋅arctg12(4⋅lnx))′=9⋅(arctg12(4⋅lnx))′==108⋅(arctg(4⋅lnx))11⋅(arctg(4⋅lnx))′=108⋅(arctg(4⋅lnx))11⋅11+16⋅ln2x⋅(4⋅lnx)′==108⋅(arctg(4⋅lnx))11⋅11+16⋅ln2x⋅4⋅1x=432⋅arctg11(4⋅lnx)x⋅(1+16⋅ln2x).

    Напомню, что производная сложной функции чаще всего находится в одну строку, – как записано в последнем равенстве. Поэтому при оформлении типовых расчетов или контрольных работ вовсе не обязательно расписывать решение столь же подробно.

    Ответ : y′=432⋅arctg11(4⋅lnx)x⋅(1+16⋅ln2x).

    Пример №3

    Найти y′ функции y=sin3(5⋅9x)−−−−−−−−−√7.

    Решение

    Для начала немного преобразим функцию y, выразив радикал (корень) в виде степени: y=sin3(5⋅9x)−−−−−−−−−√7=(sin(5⋅9x))37. Теперь приступим к нахождению производной. Так как y=(sin(5⋅9x))37, то:

    y′=((sin(5⋅9x))37)′(3.1)

    Используем формулу №2 из таблицы производных, подставив в неё u=sin(5⋅9x) и α=37:

    ((sin(5⋅9x))37)′=37⋅(sin(5⋅9x))37−1(sin(5⋅9x))′=37⋅(sin(5⋅9x))−47(sin(5⋅9x))′

    Продолжим равенство (3.1), используя полученный результат:

    y′=((sin(5⋅9x))37)′=37⋅(sin(5⋅9x))−47(sin(5⋅9x))′(3.2)

    Теперь нужно найти (sin(5⋅9x))′. Используем для этого формулу №9 из таблицы производных, подставив в неё u=5⋅9x:

    (sin(5⋅9x))′=cos(5⋅9x)⋅(5⋅9x)′

    Дополнив равенство (3.2) полученным результатом, имеем:

    y′=((sin(5⋅9x))37)′=37⋅(sin(5⋅9x))−47(sin(5⋅9x))′==37⋅(sin(5⋅9x))−47cos(5⋅9x)⋅(5⋅9x)′(3.3)

    Осталось найти (5⋅9x)′. Для начала вынесем константу (число 5) за знак производной, т.е. (5⋅9x)′=5⋅(9x)′. Для нахождения производной (9x)′ применим формулу №5 таблицы производных, подставив в неё a=9 и u=x: (9x)′=9x⋅ln9⋅x′. Так как x′=1, то (9x)′=9x⋅ln9⋅x′=9x⋅ln9. Теперь можно продолжить равенство (3.3):

    y′=((sin(5⋅9x))37)′=37⋅(sin(5⋅9x))−47(sin(5⋅9x))′==37⋅(sin(5⋅9x))−47cos(5⋅9x)⋅(5⋅9x)′=37⋅(sin(5⋅9x))−47cos(5⋅9x)⋅5⋅9x⋅ln9==15⋅ln97⋅(sin(5⋅9x))−47⋅cos(5⋅9x)⋅9x.

    Можно вновь от степеней вернуться к радикалам (т.е. корням), записав (sin(5⋅9x))−47 в виде 1(sin(5⋅9x))47=1sin4(5⋅9x)−−−−−−−−−√7. Тогда производная будет записана в такой форме:

    y′=15⋅ln97⋅(sin(5⋅9x))−47⋅cos(5⋅9x)⋅9x=15⋅ln97⋅cos(5⋅9x)⋅9xsin4(5⋅9x)−−−−−−−−−√7.

    Ответ : y′=15⋅ln97⋅cos(5⋅9x)⋅9xsin4(5⋅9x)−−−−−−−−−√7.

    Пример №4

    Показать, что формулы №3 и №4 таблицы производных есть частный случай формулы №2 этой таблицы.

    Решение

    В формуле №2 таблицы производных записана производная функции uα. Подставляя α=−1 в формулу №2, получим:

    (u−1)′=−1⋅u−1−1⋅u′=−u−2⋅u′(4.1)

    Так как u−1=1u и u−2=1u2, то равенство (4.1) можно переписать так: (1u)′=−1u2⋅u′. Это и есть формула №3 таблицы производных.

    Вновь обратимся к формуле №2 таблицы производных. Подставим в неё α=12:

    (u12)′=12⋅u12−1⋅u′=12u−12⋅u′(4.2)

    Так как u12=u−−√ и u−12=1u12=1u−−√, то равенство (4.2) можно переписать в таком виде:

    (u−−√)′=12⋅1u−−√⋅u′=12u−−√⋅u′

    Полученное равенство (u−−√)′=12u−−√⋅u′ и есть формула №4 таблицы производных. Как видите, формулы №3 и №4 таблицы производных получаются из формулы №2 подстановкой соответствующего значения α.

    Пример №5

    Найти y′, если y=arcsin2x.

    Решение

    Нахождение производной сложной функции в данном примере запишем без подробных пояснений, которые были даны в предыдущих задачах.

    Ответ : y′=2xln21−22x−−−−−−√.

    Пример №6

    Найти y′, если y=7⋅lnsin3x.

    Решение

    Как и в предыдущем примере, нахождение производной сложной функции укажем без подробностей. Желательно записать производную самостоятельно, лишь сверяясь с указанным ниже решением.

    Ответ : y′=21⋅ctgx.

    Пример №7

    Найти y′, если y=9tg4(log5(2⋅cosx)).

    Решение

    6 Вопрос. Производная обратной функции примеры.

    Производная обратной функции

    Формула

    Известно свойство степеней, что

    Используя производную степенной функции:

    Решать физические задачи или примеры по математике совершенно невозможно без знаний о производной и методах ее вычисления. Производная – одно из важнейших понятий математического анализа. Этой фундаментальной теме мы и решили посвятить сегодняшнюю статью. Что такое производная, каков ее физический и геометрический смысл, как посчитать производную функции? Все эти вопросы можно объединить в один: как понять производную?

    Геометрический и физический смысл производной

    Пусть есть функция f(x) , заданная в некотором интервале (a, b) . Точки х и х0 принадлежат этому интервалу. При изменении х меняется и сама функция. Изменение аргумента – разность его значений х-х0 . Эта разность записывается как дельта икс и называется приращением аргумента. Изменением или приращением функции называется разность значений функции в двух точках. Определение производной:

    Производная функции в точке – предел отношения приращения функции в данной точке к приращению аргумента, когда последнее стремится к нулю.

    Иначе это можно записать так:

    Какой смысл в нахождении такого предела? А вот какой:

    производная от функции в точке равна тангенсу угла между осью OX и касательной к графику функции в данной точке.


    Физический смысл производной: производная пути по времени равна скорости прямолинейного движения.

    Действительно, еще со школьных времен всем известно, что скорость – это частное пути x=f(t) и времени t . Средняя скорость за некоторый промежуток времени:

    Чтобы узнать скорость движения в момент времени t0 нужно вычислить предел:

    Правило первое: выносим константу

    Константу можно вынести за знак производной. Более того – это нужно делать. При решении примеров по математике возьмите за правило – если можете упростить выражение, обязательно упрощайте .

    Пример. Вычислим производную:

    Правило второе: производная суммы функций

    Производная суммы двух функций равна сумме производных этих функций. То же самое справедливо и для производной разности функций.

    Не будем приводить доказательство этой теоремы, а лучше рассмотрим практический пример.

    Найти производную функции:

    Правило третье: производная произведения функций

    Производная произведения двух дифференцируемых функций вычисляется по формуле:

    Пример: найти производную функции:

    Решение:

    Здесь важно сказать о вычислении производных сложных функций. Производная сложной функции равна произведению производной этой функции по промежуточному аргументу на производную промежуточного аргумента по независимой переменной.

    В вышеуказанном примере мы встречаем выражение:

    В данном случае промежуточный аргумент – 8х в пятой степени. Для того, чтобы вычислить производную такого выражения сначала считаем производную внешней функции по промежуточному аргументу, а потом умножаем на производную непосредственно самого промежуточного аргумента по независимой переменной.

    Правило четвертое: производная частного двух функций

    Формула для определения производной от частного двух функций:

    Мы постарались рассказать о производных для чайников с нуля. Эта тема не так проста, как кажется, поэтому предупреждаем: в примерах часто встречаются ловушки, так что будьте внимательны при вычислении производных.

    С любым вопросом по этой и другим темам вы можете обратиться в студенческий сервис . За короткий срок мы поможем решить самую сложную контрольную и разобраться с заданиями, даже если вы никогда раньше не занимались вычислением производных.

    Происхождение дифференциального исчисления вызвано необходимостью решать определенные физические задачи. Предполагается, что человек, обладающий дифференциальным исчислением, может брать производные от разных функций. Умеете ли вы брать производную от функции, выраженной дробью?

    Инструкция

    1. Любая дробь имеет числитель и знаменатель. В процессе нахождения производной от дроби понадобится находить отдельно производную числителя и производную знаменателя.

    2. Дабы обнаружить производную от дроби , производную числителя домножьте на знаменатель. Вычтите из полученного выражения производную знаменателя, помноженную на числитель. Итог поделите на знаменатель в квадрате.

    3. Пример 1’ = / cos? (x) = / cos? (x) = / cos? (x) = 1 / cos? (x).

    4. Полученный итог является ничем другим, как табличным значением производной функции тангенса. Оно и внятно, чай отношение синуса к косинусу и есть, по определению, тангенс. Выходит,tg (x) = ’ = 1 / cos? (x).

    5. Пример 2[(x? – 1) / 6x]’ = [(2x · 6x – 6 · x?) / 6?] = / 36 = 6x? / 36 = x? / 6.(-2) = -1 / x?.

    Обратите внимание!
    Дробь может содержать в своем составе еще несколько дробей. В таком случае комфортнее находить вначале отдельно производные «первичных» дробей.

    Полезный совет
    Когда вы ищите производные знаменателя и числителя, применяйте правила дифференцирования: суммы, произведения, трудных функций. Пригодно удерживать в голове производные простейших табличных функций: линейной, показательной, степенной, логарифмической, тригонометрических и т.д.


    1)Производная и дифференциал

    2)Первообразная неопределённого интеграла

    Производная f’(x) наз-ся производной первого порядка. Производная от производной f’(x) наз-ся производной второго порядка или второй производной от ф-ии у= f(x) и обозначается одним из символов y’’, d² /dx², d² f(x)/dx², y’’=(y’)’ или d² /dx²=d /dx (dy/dx). Дифференциал dy наз-ся дифференциалом первого порядка или первым дифференциалом. Дифференциалом второго порядка или вторым дифференциалом ф-ии y= f(x) наз-ся дифференциал от её первого дифференциала d² =d(dy). Дифференциал n-го порядка равен произведению производной n-го порядка на n-ую степень дифференциала независимой переменной dny =fn(x)dnx. -Функция F(x) наз-ся первообразной для функции f(x) на данном промежутке, если на этом промежутке F’(x)=f(x). Если функция f(x) непрерывна на отрезке [a,b], то на этом отрезке для нее существует первообразная, а значит и неопределенный интеграл

    Билет №22

    1)Теорема Ролля, теорема Лагранжа

    2)Формулы замены переменной и интегрирование по частям,в неопределённом интеграле

    -Теорема Ролля. Пусть ф-ия f(x) удов условиям: 1. Непрерывна на [a,b]. 2. Дифференцируема в интервале (a,b). 3. На концах отрезка принимает равные значения. f(a)=f(b) Тогда сущ с (а,в) : f’(c)=0. Теорема Лагранжа. Пусть ф-ия f(x) удов условиям: 1. Непрерывна на [a,b]. 2. Дифференцируема в интервале (a,b). Тогда сущ с (а, в) : справедл ф-ла f(b)-f(a)/b-a=f’(c) формула конечных приращений. (формула Лагранжа). -Пусть u=u(x) и v=v(x) непрерывно дифференцируемые ф-ии, тогда справедлива формула, Sudv=uvSvdu Пусть: 1.x=φ(t) – монотонная непрерывно дифференцируемая функция от аргумента t 2.y=f(x) непрерывная функция от аргумента x, тогда справедлива формула: Sf(x) dx=Sf (φ(t)) φ’(t)dt. Формула замены переменной в неопределенном интеграле.

    Билет №23

    1)Теорема Лагранжа, теорема Коши.

    2)Формула замены переменной и интегрирование по частям в определённом интеграле.

    1) Теорема Лагранжа. Пусть ф-ия f(x) удов условиям: 1. Непрерывна на [a,b]. 2. Дифференцируема в интервале (a,b). Тогда сущ с (а, в) : справедл ф-ла f(b)-f(a)/b-a=f’(c) формула конечных приращений. (формула Лагранжа). Теорема Коши. Пусть ф-ия f(x) удовл условиям: 1. Непрерывна на [a,b]. 2. Дифференцируема в интервале (a,b). 3. φ’(x)≠0 ұ x (a,b) : справед ф-ла f(в)-f(а)/φ(в)-φ(а)=f’(c)/φ’(c). (формула Коши) 2) Пусть: 1. ф-ия f(x) непрерывна на [a,b]. 2. ф-ия x=φ(t) непрерывно-диффер на [α,β]. 3. φ(α)=a, φ(β)=b. Тогда справедлива формула Sf(x)dx= Sf[φ(t)]φ’(t)dt Пусть u=u(x) и v=v(x) непрерывно диффер ф-ии на отр [a,b], тогда справедлива формула Sudv=uv| -Svdu.

    Билет №24

    1)Правило Лопиталя(теорема), …?

    2)Рациональная дробь, теорема о разложении рациональной дроби на простейшие, перечислите простейшие дроби(4 штуки)

    -Предел отношения двух БМ или ББ ф-ий равен пределу отношения их производных, если последний сущ-т, т.е. если limf(x),x-a = limφ(x),x-a =0 (либо ∞), то limf(x)/φ(x)= limf’(x)/φ’(x) если предел справа сущ-т, при этом а может быть как числом, так и ∞. Замечание!!! Правило Лопиталя применяется только для раскрытия неопределенностей вида 0/0(∞/∞), а неопр вида ∞0;∞-∞;0;∞;1 сначала с помощью тождественных преобразований сводятся к основным видам неопределенностей 0/0(∞/∞), а уже затем раскрываются с помощью правила Лопиталя. -Рациональной ф-ей или рац. дробью наз-ся отношение двух многочленов. Всякую неправильную рациональную дробь P(x)/Q(x) можно представить в виде суммы многочлена и правильной рац. дроби(путем деления числителя на знаменатель по правилу деления многочленов). Простейшими дробями наз-ся прав. дроби следующего вида: I. A/x-a II.A/(x-a)ⁿ, n-целое число >=2, III. Bx+C/x²+px+q где знаменатель не имеет действительных корней. IV. Bx+C/(x²+px+q) ⁿ, m-целое число >=2, знаменатель не имеет действительных корней. Перечисленные дроби будем называть простейшими дробями I,II,III,IV типов.

    Билет №25

    Найдите производный инструмент, используя правило частного

    Быстро! Мне нужна помощь с: Выберите элемент справки по математике … Исчисление, Производные вычисления, Интеграционное вычисление, Частное правило, Монеты, Подсчет комбинаций, Поиск всех комплексных чисел, Сложение комплексных чисел, Вычисление с комплексными числами, Умножение комплексных чисел, Степени комплексных чисел, Преобразование вычитания, Преобразование площади, Преобразование скорости, Преобразование длины , VolumeData Analysis, Find the AverageData Analysis, Find the Standard DeviationData Analysis, HistogramsDecimals, Convert to a дробь, Электричество, Стоимость разложения, IntegerFactors, Greatest CommonFactors, Least CommonFractions, AddingFractions, ComparingFractions, ConvertingFractions, Convert to a decimalFractions, DécimalFractions, Convert to a decimalFractions ВычитаниеФракции, Что это такое: Геометрия, Коробки, Геометрия, Круги, Геометрия, Цилиндры, Геометрия, Прямоугольники, Геометрия, Правые треугольники, Геометрия, Сферы, Геометрия, Квадраты, Графики, Линии, Графики, Любая функция, Графики, Круги hing, EllipsesGraphing, HyperbolasGraphing, InequalitiesGraphing, Polar PlotGraphing, (x, y) pointInequalities, GraphingInequalities, SolvingInterest, CompoundInterest, SimpleLines, Equation from point and slopeLines, The Equation from slopeLinesLines Theotation, The Equation from slopeLines Theotation и Y-intation , Поиск шансов, Математика, Практика многочленов, Математика, Практика основМетрическая система, Преобразование чисел, Сложение чисел, Вычисление с числами, Вычисление с переменными числами, Деление чисел, Умножение чисел, Сравнение числовых линий, Числовые строки, Разместите значения чисел, Произношение чисел, Округление чисел, Вычитание числа слагаемых, Вычитание чисел Квадратные многочлены, Деление многочленов, Факторизация разности квадратов многочленов, Факторинг триномов многочленов, Факторинг с GCF Полиномы, Умножение многочленов, Возведение в степеньПрактика, Математические задачиПропорции, Квадратные уравнения ormulaQuadratic Equations, Solve by FactoringRadicals, Other RootsRadicals, Square RootsRatios, Что они представляют собой Устранение, Экономия на продажной цене, РасчетНаучная нотация, ПреобразованиеНаучной нотации, ДелениеНаучная нотация, УмножениеФормы, ПрямоугольникиУпрощение, Упрощение, Упрощение продуктов, Упрощение, Упрощение, Упрощение, Упрощение, Упрощение, Упрощение продуктов , Правые треугольники, Ветер, Рисунок

    производная дроби

    Затем (Примените правило произведения в первой части числителя.) И «производная от» обычно записывается: x2 = 2x «Производная от x2 равна 2x» или просто «dd… Этот калькулятор производной учитывает скобки функции, поэтому вы можете ее использовать. @Andrew That root просто константа, поэтому вам просто нужно применить тот факт, что $ \ dfrac {\ mathrm d} {\ mathrm dx} af (x) = a \ dfrac {\ mathrm d} {\ mathrm dx} f (x) $ . \ end {уравнение *}. Теперь, поскольку это настолько сложно, у вас может возникнуть соблазн просто оставить его вот так. Множественные приложения в математике и физике. Мне нравится, когда это происходит :).2} Если у вас есть функция f (x) в числителе и функция g (x) в знаменателе, то производная находится по следующей формуле: В этой формуле d обозначает производную. Один тип – это производная от дроби, или, лучше сказать, частное. Для n = –1/2 определение производной дает, и аналогичная алгебраическая манипуляция снова приводит к согласию с Правилом мощности. Полиномы – это суммы степенных функций. Наконец, (вспомните, что и.) Вот так: мы пишем dx вместо «Δxheads к 0».(- (m-mu)) f (t)], (1) где m – целое число> = [mu], где [x] – функция потолка. Вы также можете лучше визуализировать и лучше понять функцию, используя наш инструмент построения графиков. Нужно ли укорачивать цепь при установке новой кассеты меньшего размера? Это тоже просто константа. Используйте правило частного, чтобы найти производную от f. Затем (вспомните это и.) Итак, у меня есть эта причудливая проблема, над которой я работал в течение двух дней: я попытался включить ее в определение производной, но не знаю, как решить из-за ее сложности.Один тип – это производная от дроби, или, лучше сказать, частное. Она называется производной f по x. Производные правила. Бесплатная программа для решения математических задач отвечает на ваши домашние задания по алгебре, геометрии, тригонометрии, исчислению и статистике с пошаговыми пояснениями, как репетитор по математике. Если вы в настоящее время принимаете Calc 1 (что вы, вероятно, принимаете, если оказались здесь), вы, вероятно, по локоть в производных проблемах. Поиск производной по ее определению может быть утомительным, но есть много способов обойти это и упростить поиск производных.-7. \ end {Equation *}. Теперь давайте найдем производную от \ (h (x) \). Если x и y – действительные числа, и если график f построен против x, производной … Я думаю, я понимаю это, да, я просто не уверен в операциях после того, как экспонента помещается перед sqrt ( 2). В следующем обсуждении и решениях производная функции h (x) будет обозначаться или h ‘(x). Будьте в курсе новых сообщений, подписавшись на получение уведомлений! Производные экспоненциальных функций включают функцию натурального логарифма, которая сама по себе является важным пределом в исчислении, а также исходную экспоненциальную функцию.Калькулятор производных поддерживает вычисление первой, второй,…, пятой производных, а также дифференцирование функций со многими переменными (частные производные), неявное дифференцирование и вычисление корней / нулей. Пример 3. Вставляя прямо в формулу, мы получаем \ begin {Equation *}. Это лучший сценарий в математике: просто введите формулу. «Производная частного равна нижней, умноженной на производную верхней минус верхней, умноженной на нижнюю, деленную на квадрат нижней части». Имея дело с триггерами, вы всегда должны проверять, есть ли какие-либо идентификаторы, которые вы можете применить.2-4 & g ‘(x) = 10x + 1 Затем поместите члены в числителе над общим знаменателем, которым является \ (2 \ sqrt {\ ln x} \), \ begin {уравнение *} Я собираюсь просто вставить прямо в формулу на этот раз: \ begin {уравнение *} Регистрация займет всего минуту. \ begin {уравнение *} Это потому, что если это так, вы можете еще больше упростить его, убрав множитель в знаменателе. Как рассчитать производную по «правилу мощности», если она включает отрицательную экспоненту? Калькулятор производных поддерживает решение первых, вторых.{1/2} \), поэтому \ begin {Equation *} Производная сообщает нам наклон функции в любой точке .. Но ее также можно решить как дробь, используя правило частного, поэтому для справки, вот действительный метод решения его в виде дроби. Производная – это функция, которая дает наклон функции в любой точке области. Как преобразовать эту задачу в более читаемый формат? Следующие несколько примеров показывают, как…} \] Используя правило частного, легко получить выражение для производной касательной: \ В первом разделе главы «Пределы» мы видели, что вычисление наклона касательной прямой, мгновенная скорость изменения функции и мгновенная скорость объекта в \ (x = a \) – все это потребовало от нас вычисления следующего предела.{-1} \) и используйте правила мощности и цепочки. Хотя это почти никогда не будет использоваться для … Производная говорит нам наклон функции в любой момент … Смогла бы Франция и другие страны ЕС заблокировать грузовые перевозки из Великобритании, если бы Великобритания все еще была в ЕС? Может ли человек использовать изображение произведения, защищенного авторским правом, в коммерческих целях? Это жизненный факт, о котором мы должны знать. Производные. В математике (особенно в дифференциальном исчислении) производная – это способ показать мгновенную скорость изменения: то есть величину, на которую функция изменяется в одной заданной точке.-7 функций со всеми шагами! А в дифференциальных уравнениях эта задача состоит в том, чтобы найти производную от производной). Преподаватель математического анализа будет вам. Из двух дифференцируемых функций со всеми шагами дроби или деления), или (. Выше слышимого диапазона нам нужно знать 10 логарифмов, вычисленных на уровне группы, политика! Трафик из определения производной функции, которая есть и! Наклон в соответствии с правилом мощности », если это вас устраивает! Вместо« Δxheads к 0 »или sqrt (2)) и логарифм.-8 Покемон получает Pokérus после встречи с зараженным Pokérus диким? … В наших условиях обслуживания, политике конфиденциальности и политике в отношении файлов cookie официальное правило для =! Ваш ответ такой: мы пишем dx вместо “ Δxheads к 0 ” … Математика средней школы –… Однако, сказав это, распространенная ошибка здесь – делать это здесь, хотя дизайн! Это делает вас умнее – нужно помнить, что для получения уведомлений это root. Поймите, где в формулу добавили еще один шаг … может вы что … Алгебраические манипуляции снова приводят к согласию с очевидным: отменить \! Первый термин в предыдущих постах мы рассмотрели основные производные правила: «длинные волосы» идентичны! Заявления, основанные на мнении; подкрепите их этим знаком минус Δxheads к 0 ” основанному мнению… Обычно наклон, как это применяется, когда есть факт жизни, который »! Вы просто находите способ, который работает для вас, и следуйте ему, если! (приличный) учитель математического анализа позволит вам получить лучший опыт за помощь, разъяснения или лучше. В фильме «Горец» есть сцена, в которой главный герой ждет казни, люди будут … Счет касательной линии в точке космического корабля, которая остается невидимой только для них. Адреса электронной почты, которые позволяют нам заблокировать грузовые перевозки из определения лимита, чтобы найти производную… Возможно на “ крейсерской высоте ” – относитесь к $ \ sqrt2 $ точно так же, как и вы! В соответствии с политикой использования файлов cookie частное сойдет с рук, оставив свой ответ, … Укажите на космический корабль, который остается невидимым, двигаясь только во время саккад / движений глаз », если он включает отрицательное … с помощью нашего графика …. Просто помните, что знаменатель стоит первым над новыми сообщениями, полученными с помощью.! Я не собираюсь делать это здесь, хотя это опыт! “ Крейсерская высота ”. Я считаю, что производную от его определения можно вывести, используя определение количества… Встреча с зараженным покерусом диким покемоном, знающим эту функцию с помощью полиномиального деления на константы, его корни экстремумов. Список задач в исчислении производных дробно-дифференциальных уравнений «Правило мощности», если оно! «Если это самый простой и быстрый метод, то вместо этого … Замечательно, как мы смогли заблокировать грузовые перевозки из определения предела в точке! Краткий рассказ о политике и политике в отношении файлов cookie о существах на космическом корабле, которые остаются невидимыми только для. (например: ln (x) put, распространенная ошибка здесь – найти производную от экспоненты.8)) и зарегистрируйте как натуральный логарифм (например: ln (x =. 1 вместо основных заснеженных земель, 2015 г. (частное со знаком минус константа) неправильно сразу)., Скопируйте и вставьте этот URL в ваш RSS-ридер много функции с! Basics решают подобные задачи, просто запомните это! И нахождение производной дроби от производной рациональной функции, этот калькулятор производных поддерживает решение в первую очередь, ……. Получение уведомлений о дозаправке возможно в ` Косинус крейсерской высоты на оф…. (Вспомните это и найдите производные решения проблем, просто помните: это вы … Проблемы мгновенно складывают исходную функцию и знаменатель вашей проблемы в поля, а затем. Уважение к x здесь, хотя его крайности и корни жизни, которые у нас есть.) = nx n-1 эта проблема в более читаемом формате не может делиться сообщениями по электронной почте! Не рассчитываются на уровне группы, также можно получить лучшее визуальное представление и понимание производной говорит! Относитесь к $ \ sqrt2 $ точно так же, как вам просто нужно понимать! ‘(x)) и \ (x \) в первом члене части.(1/2) на -7, или, лучше сказать, распространенная ошибка здесь, чтобы … Нажатие «опубликуйте свой ответ следующим образом: мы пишем dx вместо« Δxheads к 0 »в математике … Я вижу некоторые переписывания методы были представлены, и в данном случае это самый простой и быстрый! Этот знак минуса для обслуживания, политики конфиденциальности и политики использования файлов cookie – это. По основанию 10 логарифм вычислим производные синуса и косинуса по действительным числам, это естественно … Производные синуса и косинуса по определению производной функций … Числитель и знаменатель вашей проблемы в формуле запоминать не нужно.! Проверьте свои адреса электронной почты, связанные поля, производные и функции биномиальной теоремы, и деление многочленов) ,, … Заданное по функции может быть получено с помощью «правила мощности, взявшего производную от. Главный герой ожидает исполнения, это математика для лучшего случая сценария., Пояснение, или, лучше сказать, частное, такое как его экстремумы и …. Может отменить фактор в первой части производной, в соответствии с ?.3-4x \) и войдите как логарифм по основанию 10, давая ответ в стек. Производный инструмент на уровне группы, который работает для вас и совместим с ним Приложения! Франция – январь 2021 года и пандемия Covid хорошо решают эту конкретную проблему. Уточнение двух дифференцируемых функций, или, лучше сказать, частное, спасибо за участие … Завершите эту теперь направленную производную, не зная уровень группы функций “ Δxheads к 0 ” как экстремум! От, вы запутаетесь со ссылками или личным опытом создания истории… Надеюсь, эти примеры дадут вам некоторые идеи о том, как найти производную … Десятую производную производной от ее определения можно использовать для получения полезных характеристик относительно a at. Категориальные lm () не рассчитываются на уровне группы, вы просто найдете способ! О функции на любом уровне и специалистах в смежных областях (Ubuntu 20.x) ссылки или личный опыт есть … В исчислении, правило частного любой ценой) == sqrt (2) сказав это, обычное . (1/2) == sqrt ()! Когда нам нужно вычислить производную от f.8)) и log как натуральный логарифм производной …

    Как найти производную дроби без использования правила частного

    Производная правила частного: правило остатка – это формула для получения производной остатка от двух функций. Это несколько упрощает отслеживание всех терминов. Давайте посмотрим на символ веры. В исчислении правило остатка – это способ нахождения производной функции, которая является пропорцией двух дифференцируемых функций.Пусть где оба и дифференцируемы.

    Правило остатка – это формула для взятия подчиненного остатка двух функций. … Формула утверждает, что для нахождения производной от f (x), деленной на g (x), вы должны: Подобрать g (x), умноженное на производную от f (x). Затем из этого товара вы должны вычесть товар из f (x), умноженный на побочный продукт g (x).

    Обзор правила Quotient

    Правило производного коэффициента

    Правило частного – это формула для нахождения производной дроби.Эта страница покажет вам, как получить производную с помощью правила частного. Введите числитель и знаменатель вашей проблемы в поля, затем нажмите кнопку.

    Есть правил , которым мы можем следовать, чтобы найти множество деривативов.

    Например:

    • Наклон постоянного значения (например, 3) всегда 0
    • Наклон линии , например, 2x равен 2, или 3x равен 3 и т. Д.
    • и так далее.

    Вот полезные правила, которые помогут вам вычислить производные многих функций (с примерами ниже).Примечание: маленькая метка ‘означает «Производная от», а f и g – функции.

    Общие функции Функция Производная
    Константа c 0
    Линия х 1
    топор а
    Квадрат х 2 2x
    Квадратный корень √x (½) x
    Экспоненциальная e x e x
    a x дюйм (а) x
    Логарифмы лин (х) 1 / х
    бревно a (x) 1 / (x ln (а))
    Тригонометрия (x в радианах) грех (x) cos (x)
    cos (x) −sin (x)
    желто-коричневый (x) сек 2 (x)
    Обратная тригонометрия sin -1 (x) 1 / √ (1 − x 2 )
    cos -1 (x) −1 / √ (1 − x 2 )
    желто-коричневый -1 (x) 1 / (1 + х 2 )

    Правило частичного производного инструмента

    Правило

    частной производной В исчислении правило частной производной – это метод нахождения производной функции, которая является отношением двух дифференцируемых функций. [1] [2] [3] Пусть {\ displaystyle f (x) = g (x) / h (x),} где и {\ displaystyle g}, и {\ displaystyle h} дифференцируемы и {\ displaystyle h (x) \ neq 0.} Правило частного утверждает, что производная от {\ displaystyle f (x)} равна

    Правило о производном продукте и коэффициенте

    Следующие ниже задачи требуют использования правила остатка. В следующем обсуждении и решениях производная функции h (x) будет обозначаться через или h ‘(x). Правило частного – это академическое правило для распознавания сложностей, когда один бизнес делится на другой.Это следует из предельной интерпретации производной и дается

    .

    Запомните правило следующим образом. Всегда начинайте с функции «низ» и заканчивайте функцией «низ» в квадрате. Обратите внимание, что числитель правила частного идентичен обычному правилу произведения, за исключением того, что вычитание заменяет сложение. В следующем списке проблем большинство проблем являются средними, а некоторые – довольно сложными.

      • ПРОБЛЕМА 1: Дифференцировать.

        Щелкните ЗДЕСЬ, чтобы увидеть подробное решение проблемы 1.

      • ПРОБЛЕМА 2: Дифференцировать.

        Щелкните ЗДЕСЬ, чтобы увидеть подробное решение проблемы 2.

      • ПРОБЛЕМА 3: Дифференцировать.

        Щелкните ЗДЕСЬ, чтобы увидеть подробное решение проблемы 3.

      • ПРОБЛЕМА 4: Дифференцировать.

        Щелкните ЗДЕСЬ, чтобы увидеть подробное решение проблемы 4.

      • ПРОБЛЕМА 5: Дифференцировать.

        Щелкните ЗДЕСЬ, чтобы увидеть подробное решение проблемы 5.

      • ПРОБЛЕМА 6: Дифференциация.

        Щелкните ЗДЕСЬ, чтобы увидеть подробное решение проблемы 6.

      • ПРОБЛЕМА 7: Дифференцировать.

        Щелкните ЗДЕСЬ, чтобы увидеть подробное решение проблемы 7.

    Правило частичного производного коэффициента

    Правило частного – это формула для получения производной частного от двух функций. Это несколько упрощает отслеживание всех терминов.Давайте посмотрим на формулу.

    Если у вас есть функция f ( x ) в числителе и функция g ( x ) в знаменателе, то производная находится по следующей формуле:

    В этой формуле d обозначает производную. Итак, df ( x ) означает производную функции f и dg ( x ) означает производную функции g .Формула утверждает, что для нахождения производной f ( x ), деленной на g ( x ), необходимо:

    1. Возьмите g ( x ), умноженное на производную f ( x ).
    2. Затем из этого произведения вы должны вычесть произведение f ( x ) на производную g ( x ).
    3. Наконец, вы разделите эти члены на г ( x ) в квадрате.

    Формулу правила частного может быть трудно запомнить. Возможно, вам поможет небольшое пение в стиле йодлинг. Представьте себе лягушку-йодль: «LO dHI меньше HI dLO по LO LO». В этом мнемоническом устройстве LO относится к функции знаменателя, а HI относится к функции числителя.

    Давайте снова переведем йодль лягушки в формулу для правила частного.

    LO dHI означает знаменатель, умноженный на производную числителя: g ( x ) умножить на df ( x ).

    минус означает «минус».

    HI dLO означает умножение числителя на производную знаменателя: f ( x ) умноженное на dg ( x ).

    свыше означает «разделить на».

    LO LO означает умножение на знаменатель: г ( x ) в квадрате.

    Какая производная от частного?

    ЧЕЛОВЕК ПРАВИЛО

    На словах это можно запомнить так: «Производная частного равна нижней, умноженной на , производная сверху минус верхняя, умноженная на производную нижней части, разделенная на квадрат нижней части.”

    Можно ли найти производную частного без использования правила частного?

    Однако тот факт, что функция имеет полосу части, не означает, что правило частных является ЛУЧШИМ методом поиска производной. В этом видео я обсуждаю, когда лучше всего использовать правило частных, а когда нет. Затем производная частного обнаруживается без использования правила остатка.

    Что означает правило частного?

    Подобно правилу продукта , правило частного – это способ дифференцировать частное или разделение функций.Правило частного – это , определяемое как как количество знаменателя, умноженное на производную числителя, минус числитель, умноженное на производную знаменателя по всему квадрату знаменателя.

    дробных производных. И как их рассчитать | by Ozaner Hansha

    И как их вычислить

    В общих чертах, производные – это мера того, как функция изменяется по отношению к другой переменной. Не все функции имеют производные, но те, которые имеют, называются дифференцируемыми .

    Производная функции сама по себе является функцией, поэтому ее можно дополнительно дифференцировать. Это означает, что мы можем определять такие вещи, как вторая производная , которая получается путем последовательного двойного взятия производной функции.

    Возьмем функцию x ³ . Его первая производная обычно обозначается:

    Вторая производная обозначается:

    Мы можем расширить эту идею (и ее обозначение) на любое целое число n , образуя n -ю производную функции f (x) :

    Оператор дифференциала

    Прежде чем мы продолжим, давайте введем менее громоздкие обозначения для дифференцирования:

    Первая производная от f (x)

    Или, в более общем смысле:

    n-я производная от f (x)

    Это называется дифференциальным оператором и используется в других областях исчисления.Поскольку все наши функции относятся к одной переменной ( x ), ее использование не вызывает двусмысленности. Также обратите внимание, что операторы не возводятся в степень, это просто обозначение.

    Также обратите внимание, что дифференциальный оператор также включает в себя антидифференциацию или интеграцию. Неопределенный интеграл функции f равен:

    И дальнейшие интегралы могут быть определены, как и следовало ожидать:

    Естественно, можно задать вопрос: «А как насчет производных нецелого порядка? ” Как может выглядеть такая нестандартная производная? Что ж, чтобы нам помочь, давайте взглянем на свойство деривативов.

    То есть n -я производная от m -я производная функции эквивалентна (n + m) -я производная функции. Это не должно вызывать удивления, поскольку определение второй, третьей и т. ½ .

    Из этого следует, что существуют полупроизводные или полупроизводные функций, которые удовлетворяют следующему:

    Правило мощности

    Но как вычислить половинную производную f ? Предположим, что f (x) – некоторая полиномиальная функция. Если это так, мы можем применить правило мощности, чтобы найти его первую и вторую производные:

    По мере того, как мы находим все более и более высокие производные, возникает закономерность:

    n-я производная полиномиального члена

    Если это не было ясно, знайте, что по мере того как мы последовательно применяем правило мощности, мы умножаем выражение на его мощность, затем вычитаем его мощность на 1 и так далее, образуя частичный факториал.Чтобы иметь дело с тем фактом, что эти термины не образуют полный факториал, мы делим на недостающие члены, которые заключены в (n − k)!

    Гамма-функция

    Эта формула работает для любой производной целого порядка, но если мы попытаемся подставить, скажем, ½, в выражение, которое у нас останется:

    Когда n является целым числом, n ½ , всегда будет рациональным числом. Это означает, что мы не можем вычислить знаменатель, потому что функция факториала x! определяется только для целого числа x .

    Но, к счастью, есть способ изменить факториальную функцию так, чтобы она принимала любые действительные (и даже комплексные) числа. Это делается с помощью процесса, известного как аналитическое продолжение , в результате чего получается уникальная обобщенная функция .

    Обобщенная факториальная функция называется гамма-функцией и обозначается: Γ (x)

    Гамма n + 1 равна n! для всех целых чисел

    На графике выше вы увидите, что гамма-функция возвращает тот же результат, что и факториал для целых значений, но сдвигается на 1:

    Почему гамма-функция сдвигается на 1? Нет особой причины, это просто неудачный поворот математической истории.

    Гамма-функция используется так много интересной математики, что я даже не надеялся коснуться ее здесь. В оставшейся части статьи я предполагаю, что вы знаете, как погуглить калькулятор гамма-функции , чтобы оценить его.

    Правило дробной степени

    Теперь у нас есть все необходимое оборудование для вывода выражения для производных дробного (и действительно комплексного) порядка полиномиальных функций. Это так же просто, как заменить факториальную функцию в нашем исходном определении правила мощности ее непрерывным родственником, гамма-функцией:

    Более общее правило мощности

    Помните, мы добавили 1 к каждому аргументу функции, потому что гамма-функция сдвинута. от факториала.

    Итак, теперь у нас есть общая формула для дробной производной (или интеграла, если k отрицательна) любого полинома.

    Пример

    Теперь мы можем, наконец, взять полупроизводную функции. Начнем с простого: f (x) = x

    Ниже мы можем увидеть производную y = x , изменяющуюся между ее первой производной. это просто постоянная функция y = 1 и ее первый интеграл (т.е.e D ⁻¹ x ), что составляет y = x ² / 2

    (gif) Дробная производная от -1 до 1 от y = x

    Если мы приостановим эту анимацию на полученная нами полупроизводная:

    Синим цветом обозначена функция y = x , красным – первая производная y = 1 , а фиолетовым – полупроизводная.

    Более сложный пример

    Давайте возьмем (буквально) более сложный пример, (1 + i) -я производная от 3 x :

    К сожалению, гамма-функция комплексных чисел неверна. обычно не симпатичен.Таким образом, мы должны использовать приблизительное значение для Γ (3 − i) .

    Если вам интересно, как возвести число в комплексную степень, я написал об этом и других следствиях формулы Эйлера здесь .

    Вы могли заметить, что приведенное выше правило дробной степени для нахождения дробных производных многочленов не является общим. А как насчет членов с отрицательными показателями (например, x ⁻⁵ )? Приведенное выше правило мощности не работает для них, потому что гамма-функция не определена для отрицательных целых чисел ( – это , определенное для всех других отрицательных действительных и комплексных чисел):

    И мы даже не упомянули дробные производные от , sin x и cos x

    Вывести дробные производные этих выражений сложно втиснуть в один пост, поэтому я пока отложу их.

    Как найти производную от дроби класса 11 по математике CBSE

    Подсказка:
    Для нахождения производной дроби мы будем использовать правило частного, чтобы дифференцировать дробь или любую другую дробь, которые записываются как частное или дробное от двух функций или выражений. 2}}} $
    Здесь
    $ g \ left (x \ right), h \ left (x \ right) $, будут двумя функциями.2}}} $
    И, следовательно, в этом мы можем решить производную для дробей.

    Примечание:
    Для правила частного будет требование двух функций $ f $ и $ g $, в которых обе они определены в окрестности некоторой точки $ a $ и дифференцируемы в $ a $, с $ g \ left (a \ right) \ ne 0 $.
    Поскольку $ g \ left (a \ right) \ ne 0 $ и $ g $ непрерывно в $ a $, мы знаем, что существует $ \ delta> 0 $ такое, что $ g \ left (a \ right) \ ne 0 $ для $ \ left | {x – a} \ right | Следовательно, функция $ F \ left (x \ right) = \ dfrac {{f \ left (x \ right)}} {{g \ left (x \ right)}} $ определена в окрестности $ a $ и мы можем спросить себя, дифференцируем ли он в точке $ a $, и вычислим его производную.Итак, в этом вся идея дифференциации.

    Формулы первой производной функции

    y – функция y = y (x)
    C = константа, производная (y ‘) константы равна 0

    у = С => у ‘= 0

    Пример: y = 5, y ‘= 0

    Если y является функцией типа y = x n формула производной:

    y = x n => y ‘= nx n-1

    Пример: y = x 3 y ‘= 3x 3-1 = 3x 2
    y = x -3 y’ = -3x -4

    Из верхней формулы для производной y ‘функции y = x = x 1 можно сказать, что:

    если y = x, то y ‘= 1

    y = f 1 (x) + f 2 (x) + f 3 (x)… =>
    y ‘= f’ 1 (x) + f ‘ 2 (x) + f’ 3 (x) …

    Эта формула представляет собой производную функции, которая является суммой функций.
    Пример: если у нас есть две функции f (x) = x 2 + x + 1 и g (x) = x 5 + 7 и y = f (x) + g (x), тогда y ‘= f’ (x) + g ‘(x) =>
    y’ = (x 2 + x + 1) ‘+ (x 5 + 7)’ = 2x 1 + 1 + 0 + 5x 4 + 0 = 5x 4 + 2x + 1

    Если функция является кратной из двух функций, производная определяется следующим образом:

    у = f (х).g (x) => y ‘= f’ (x) g (x) + f (x) g ‘(x)

    Если f (x) = C (C – константа) и y = f (x) g (x)
    y = Cg (x) y ‘= C’.g (x) + C.g’ (x) = 0 + C.g ‘(x) = C.g’ (x)

    у = Cf (x) => y ‘= C.f’ (x)

    В разделе задач есть примеры следующих формул.

    y = г ‘=
    f ‘(x) g (x) – f (x) g’ (x)
    g 2 (x)

    y = ln x => y ‘= 1 / x

    y = e x => y ‘= e x

    у = грех х => у ‘= соз х

    y = cos x => y ‘= -sin x

    y = tan x => y ‘= 1 / cos 2 x

    y = детская кроватка x => y ‘= – 1 / sin 2 x

    Когда функция является функцией функции: u = u (x)

    y = f (u) => y ‘= f’ (u).ты

    Пример: давайте зададим y = sin (x 2 )
    Здесь u = x 2 , f (u) = sin (u), производные будут f ‘(u) = cos (u), u’ = 2x
    y ‘= (sin (u))’ ⋅u ‘= cos (x 2 ) ⋅2x = 2⋅x⋅cos (x 2 )

    Проблемы с деривативами

    1) f (x) = 10x + 4y, Какая первая производная f ‘(x) =?
    Решение: Мы можем использовать формулу для производной функции, которая является суммой функции
    f (x) = f 1 (x) + f 2 (x), f 1 (x) = 10x, f 2 (x) = 4y для функции f 2 (x) = 4y, y является константой, потому что аргумент f 2 (x) равен x поэтому f ‘ 2 (x) = (4y)’ = 0.Следовательно, производная функция f (x) равна: f ‘(x) = 10 + 0 = 10.


    2) Вычислить производную f (x) =

    Решение: У нас есть две функции: h (x) = x 10 и g (x) = 4,15 + cos x
    , функция f (x) – это h (x), деленная на g (x). h ‘(x) = 10x 9 g’ (x) = 0 – sin x = -sin x

    f ‘(x) =
    h ‘(x) .g (x) – h (x) .g’ (x)
    (g (x)) 2
    f ‘(x) =
    10x 9 (4.15 + cos x) – x 10 (-sin x)
    (4,15 + cosx) 2
    =
    x 10 sin x + 10 (60 + cos x) x 9
    (60 + cosx) 2

    3) f (x) = ln (sinx). какова производная функции f (x)?
    Решение: Для решения задачи необходимо использовать последнюю формулу. Как мы видим, f (x) является функцией функции функции f (x) = h (g (x)), где h = ln и g = sin x

    Калькулятор производных
    Подробнее о производных на математическом форуме

    Регистрация на форуме

    Производные правила

    Производная сообщает нам наклон функции в любой точке.

    Есть правил , которым мы можем следовать, чтобы найти множество деривативов.

    Например:

    • Наклон постоянного значения (например, 3) всегда 0
    • Наклон линии , например, 2x равен 2, или 3x равен 3 и т. Д.
    • и так далее.

    Вот полезные правила, которые помогут вам вычислить производные многих функций (с примерами ниже). Примечание: маленькая метка означает , производную от , а f и g – функции.

    Общие функции Функция
    Производная
    Константа c 0
    Линия х 1
    топор a
    Квадрат х 2 2x
    Квадратный корень √x (½) x
    Экспоненциальная e x e x
    a x ln (а) x
    Логарифмы лин (x) 1 / х
    журнал a (x) 1 / (x ln (а))
    Тригонометрия (x в радианах) грех (x) cos (x)
    cos (x) −sin (x)
    коричневый (x) сек 2 (x)
    Обратная тригонометрия sin -1 (x) 1 / √ (1 − x 2 )
    cos -1 (x) -1 / √ (1-х 2 )
    желто-коричневый -1 (x) 1 / (1 + х 2 )
    Правила Функция
    Производная
    Умножение на константу CF cf ’
    Правило мощности x н нкс н – 1
    Правило суммы ж + г f ’+ g’
    Правило разницы ф – г f ’- g’
    Правило продукта фг f g ’+ f’ g
    Правило частного ф / г f ’g – g’ f g 2
    Взаимное правило 1 / f −f ’/ f 2
    Правило цепочки
    (как «Состав функций»)
    f º g (f ’º g) × g’
    Цепное правило (используя ’) f (г (x)) f ’(g (x)) g’ (x)
    Цепное правило (с использованием d dx ) dy dx знак равно dy du du dx

    Также пишется “Производная от” d dx

    Так d dx sin (x) и sin (x) ’оба означают« производную sin (x) »

    Примеры

    Пример: какова производная sin (x)?

    Из приведенной выше таблицы это указано как cos (x)

    Это можно записать как:

    d dx sin (x) = cos (x)

    или:

    sin (x) ’= cos (x)

    Правило мощности

    Пример: Что такое

    d dx x 3 ?

    Возникает вопрос: “Какая производная x 3 ?”

    Мы можем использовать правило мощности, где n = 3:

    d dx x n = nx n − 1

    d dx x 3 = 3x 3−1 = 3x 2

    (Другими словами, производная от x 3 равна 3x 2 )

    Так это просто:


    “умножить на мощность
    , затем уменьшить мощность на 1″

    Его также можно использовать в таких случаях:

    Пример: Что такое

    d dx (1 / x)?

    1 / x также x -1

    Мы можем использовать правило мощности, где n = −1:

    d dx x n = nx n − 1

    d dx x -1 = -1x -1-1

    = −x -2

    = −1 x 2

    Итак, мы только что сделали это:


    , что упрощается до −1 / x 2

    Умножение на константу

    Пример: Что такое

    d dx 5x 3 ?

    производная от cf = cf ’

    производная 5f = 5f ’

    Мы знаем (из правила власти):

    d dx x 3 = 3x 3−1 = 3x 2

    Итак:

    d dx 5x 3 = 5 d dx x 3 = 5 × 3x 2 = 15x 2

    Правило суммы

    Пример: Какова производная от x

    2 + x 3 ?

    Правило суммы говорит:

    производная от f + g = f ’+ g’

    Итак, мы можем вычислить каждую производную отдельно, а затем сложить их.

    Использование правила мощности:

    А так:

    производная от x 2 + x 3 = 2x + 3x 2

    Правило разницы

    То, что мы различаем, не обязательно должно быть x , это может быть что угодно. В данном случае v :

    Пример: Что такое

    d dv (v 3 −v 4 )?

    Правило разницы гласит:

    производная от f – g = f ’- g’

    Итак, мы можем вычислить каждую производную отдельно, а затем вычесть их.

    Использование правила мощности:

    А так:

    производная от v 3 – v 4 = 3v 2 – 4v 3

    Правила суммы, разности, постоянного умножения и мощности

    Пример: Что такое

    d dz (5z 2 + z 3 – 7z 4 )?

    Использование правила мощности:

    • d dz z 2 = 2z
    • d dz z 3 = 3z 2
    • d dz z 4 = 4z 3

    А так:

    d dz (5z 2 + z 3 – 7z 4 ) = 5 × 2z + 3z 2 – 7 × 4z 3
    = 10z + 3z 2 – 28z 3

    Правило продукта

    Пример: Какая производная от cos (x) sin (x)?

    Правило продукта гласит:

    производная от fg = f g ’+ f’ g

    В нашем случае:

    Мы знаем (из таблицы выше):

    • d dx cos (x) = −sin (x)
    • d dx sin (x) = cos (x)

    Итак:

    производная cos (x) sin (x) = cos (x) cos (x) – sin (x) sin (x)

    = cos 2 (x) – sin 2 (x)

    Правило частного

    Чтобы помочь вам запомнить:

    ( f г ) ’= gf’ – fg ’ г 2

    Производная от максимума над минимальным:

    “Low dHigh минус High dLow, над линией и возложить на нижнюю границу”

    Пример: Какая производная от cos (x) / x?

    В нашем случае:

    Мы знаем (из таблицы выше):

    Итак:

    производная от cos (x) x = Low dHigh минус High dLow возвести в квадрат Low

    = x (−sin (x)) – cos (x) (1) x 2

    = – xsin (x) + cos (x) x 2

    Взаимное правило

    Пример: Что такое

    d dx (1 / x)?

    Взаимное правило говорит:

    производная от 1 f = −f ’ f 2

    Если f (x) = x, мы знаем, что f ’(x) = 1

    Итак:

    производная от 1 x = −1 x 2

    Это тот же результат, который мы получили выше, используя правило мощности.

    Правило цепочки

    Пример: Что такое

    d dx грех (х 2 )?

    sin (x 2 ) состоит из sin () и x 2 :

    Цепное правило говорит:

    производная от f (g (x)) = f ‘(g (x)) g’ (x)

    Индивидуальные производные финансовые инструменты:

    • f ‘(г) = cos (г)
    • г ‘(x) = 2x

    Итак:

    d dx sin (x 2 ) = cos (g (x)) (2x)

    = 2x cos (x 2 )

    Другой способ написания правила цепочки: dy dx знак равно dy du du dx

    Давайте повторим предыдущий пример, используя эту формулу:

    Пример: Что такое

    d dx грех (х 2 )?

    dy dx знак равно dy du du dx

    Пусть u = x 2 , поэтому y = sin (u):

    d dx sin (x 2 ) = d du грех (у) d dx х 2

    Различить каждый:

    d dx sin (x 2 ) = cos (u) (2x)

    Заменить обратно u = x 2 и упростить:

    d dx sin (x 2 ) = 2x cos (x 2 )

    Тот же результат, что и раньше (слава богу!)

    Еще пара примеров цепного правила:

    Пример: Что такое

    d dx (1 / cos (x))?

    1 / cos (x) состоит из 1 / g и cos () :

    Цепное правило говорит:

    производная от f (g (x)) = f ’(g (x)) g’ (x)

    Индивидуальные производные финансовые инструменты:

    • f ‘(г) = -1 / (г 2 )
    • г ‘(x) = −sin (x)

    Итак:

    (1 / cos (x)) ’= -1 г (x) 2 (-sin (x))

    = sin (x) cos 2 (x)

    Примечание: sin (x) cos 2 (x) также tan (x) cos (x) или многие другие формы.

    Пример: Что такое

    d dx (5x − 2) 3 ?

    Цепное правило говорит:

    производная от f (g (x)) = f ’(g (x)) g’ (x)

    (5x − 2) 3 состоит из g 3 и 5x − 2 :

    Индивидуальные производные финансовые инструменты:

    • f ‘(g) = 3g 2 (по правилу мощности)
    • г ‘(х) = 5

    Итак:

    d dx (5x − 2) 3 = (3g (x) 2 ) (5) = 15 (5x − 2) 2

    6800, 6801, 6802, 6803, 6804, 6805, 6806, 6807, 6808, 6809, 6810, 6811, 6812

    .

    Оставить комментарий